You are on page 1of 104

1

. 15,16,17
Autopsy of a 70-year-old woman reveals a papillary growth within the left ventricular chamber. The
growth
consists of a small mass of finger-like projections attached to the mitral valve, without associated
valvular or other
cardiac abnormalities. Histologically, each papillary structure is composed of a core of fibrous tissue
lined by
thickened endothelium. The patient did not have any history of cardiac disease or evidence of
thromboembolism.
Which of the following is the most likely diagnosis?

A. Acute mural thrombus

B. Cardiac myxoma

C. Infective endocarditis

D. Nonbacterial thrombotic endocarditis

E. Papillary fibroelastoma

Explanation:

The correct answer is E. Gross and microscopic features of this small mass in the left ventricle are
entirely
consistent with papillary fibroelastoma. This lesion is not neoplastic, despite the sound of the name. It
probably
results from organized thrombi forming on the endocardial surfaces of the mitral valve or left
ventricular cavity.
Papillary fibroelastomas are usually clinically silent and are discovered at autopsy as an incidental
finding.

An acute mural thrombus (choice A) would not have a core of fibrous tissue. Acute mural thrombosis
usually
develops as a result of stasis in the ventricular cavities, in association with ventricular enlargement,
myocardial
infarction, or ventricular aneurysm, for example. Thrombosis often develops in the atria when there is
atrial
fibrillation.

Cardiac myxoma (choice B) is the most frequent primary cardiac neoplasm. It is benign and consists
of stellate
mesenchymal cells within a myxoid background. Since the left atrium is the most frequent location,
this tumor
can produce mitral stenosis by a ball-valve effect.

Both forms of endocarditis are associated with formation of vegetations attached to the surface of the
atrioventricular valves. Vegetations of infective endocarditis (choice C) are bulky and composed of
fibrin,
bacteria, and inflammatory cells. Since nonbacterial thrombotic endocarditis (choice D) is caused by
hypercoagulable states, the vegetations consist of aggregates of fibrin but few inflammatory cells and
no
bacteria.
2

Note that all of the above conditions may lead to systemic embolization. Fragments of vegetations,
thrombi,
myxoma, and papillary fibroelastoma may detach and be released into the bloodstream, causing
infarcts.

A 45-year-old man presents to his physician with hematuria. Renal biopsy demonstrates a focal necrotizing
glomerulitis with crescent formation. The patient has a history of intermittent hemoptysis and
intermittent chest
pain of moderate intensity. A previous chest x-ray had demonstrated multiple opacities, some of which
were
cavitated. The patient also has chronic cold-like nasal symptoms. Which of the following is the most
likely
diagnosis?

A. Aspergillosis

B. Polyarteritis nodosa

C. Renal carcinoma metastatic to the lung

D. Tuberculosis

E. Wegener's granulomatosis

Explanation:

The correct answer is E. While in real life, other diseases (or combinations of diseases) may
occasionally cause
concurrent pulmonary, sinus, and renal involvement, if you see this pattern on a test question, you
should
immediately think of Wegener's granulomatosis. This is a rare focal necrotizing vasculitis of still
unclear etiology,
which also features prominent granulomas, some of which are centered on the vascular lesions. The
vasculitis
and granulomas can involve the entire respiratory tract, and an easily obtained nasal biopsy may
sometimes
yield the diagnosis. The renal involvement is usually in the form of a necrotizing glomerulonephritis.
The
disease typically affects middle aged men, and its formerly poor prognosis has been improved by
corticosteroid
and cyclophosphamide therapy.

Aspergillosis (choice A) can cause prominent lung disease, but does not usually have renal
involvement.

Polyarteritis nodosa (choice B) is a possibility, but usually spares the lungs.

Renal cell carcinoma (choice C) might cause lung nodules, but there is no evidence of tumor in the
kidney.

Tuberculosis (choice D) can cause prominent lung disease, but does not usually have renal
involvement.
3

A 55-year-old man presents to a physician with jaundice. Ultrasonography demonstrates a 5 cm mass in the
head
of the pancreas. Endoscopic retrograde cholangiopancreatography with cytologic sampling
demonstrates cells
with large hyperchromatic nuclei and a high nuclear/cytoplasmic ratio. A few small glands composed
of these cells
are also seen in the cytologic preparation. The overall prognosis for this man will be most similar to
that of a
patient with which of the following malignancies?

A. Adenocarcinoma of the breast

B. Adenocarcinoma of the colon

C. Adenocarcinoma of the esophagus

D. Adenocarcinoma of the prostate

E. Primary gastric lymphoma

Explanation:

The correct answer is C. The patient probably has pancreatic adenocarcinoma. This cancer carries one
of the
worst prognoses, with a 3.5% overall 5 year survival rate despite all attempts at aggressive
management. The
prognosis is also bleak with adenocarcinoma of the esophagus, with a 10% overall 5 year survival
rate.

Adenocarcinoma of the breast (choice A) now has an overall 5 year survival rate of 60-70%.

Adenocarcinoma of the colon (choice B) now has an overall 5 year survival rate of 50-60%.

Adenocarcinoma of the prostate (choice D) now has an overall 5 year survival rate of 50-70%.

Primary gastric lymphoma (choice E) has an overall survival 5 year survival rate of 75-85%.

Biopsy of a persistent exophytic area on the vulva of a 60-year-old woman demonstrates a squamous
epithelial
lesion. No koilocytes are seen. The lesions show papillary projections composed of disordered,
squamous
epithelium with well-differentiated cells. The basement membrane at the dermal-epidermal junction is
focally
disrupted by squamous cell groups extending deep into the dermis. Which of the following diagnoses
is most
accurate?

A. Condyloma acuminatum
4

B. Extramammary Paget's disease

C. Vulvar intraepithelial neoplasia

D. Vulvar melanoma

E. Vulvar squamous cell carcinoma

Explanation:

The correct answer is E. The disruption of the basement membrane with cell groups extending deep
into the
dermis indicates that this is invasive squamous cell carcinoma of the vulva, which may arise in vulvar
intraepithelial neoplasia, in condyloma, or spontaneously. The latter type (which is likely here) tends
to be well
differentiated, while the former two are often poorly differentiated.

Condyloma acuminatum (choice A) would contain koilocytes and would not cross the basement
membrane.

Extramammary Paget's disease (choice B) would probably contain individual tumor cells that stain for
mucin.

The lesion of vulvar intraepithelial neoplasia (choice C) does not cross the basement membrane.

Vulvar melanoma (choice D) is composed of melanocytes that would mark with S100 or HMB-45.

Which of the following conditions is associated with overexpression of bcl-2?

A. Acute lymphoblastic leukemia

B. Burkitt lymphoma

C. Follicular lymphoma

D. Multiple myeloma

E. Small lymphocytic lymphoma

Explanation:

The correct answer is C.bcl-2 inhibits apoptosis; a (14;18) translocation resulting in overexpression of
the bcl-2
protein in B lymphocytes causes apoptosis of neoplastic cells to be permanently inhibited, producing
follicular
lymphoma. Follicular lymphoma involves the lymph nodes and bone marrow of older people and is
characterized by slow, but relentless, growth. Thus, this lymphoma allows a long survival even
without any
treatment. Indeed, follicular lymphoma is resistant to aggressive therapy because of its low mitotic
activity.
5

Acute lymphoblastic leukemia (ALL) (choice A) is a neoplasm of pre-B cells that affects children.
Different
molecular mechanisms probably account for different cases. Hyperdiploidy, the Philadelphia
chromosome,
t(12;21), and t(4;11) are variably found in patients with ALL.

Burkitt lymphoma (choice B) occurs in different clinical settings, including the African endemic,
sporadic
nonendemic, and AIDS-associated forms. Histologically, all these forms are identical and are derived
from
neoplastic transformation of B cells expressing surface IgM. Common to all forms are translocations
involving
the c-myc oncogene (chromosome 8) and Ig loci (chromosome 14). These translocations lead to
overexpression of c-myc, which encodes a nuclear transcription protein, resulting in sustained cell
replication.
The African form is associated with Epstein-Barr virus infection.

Multiple myeloma (choice D) is a neoplasm of terminally differentiated B lymphocytes that secrete


large
amounts of monoclonal Ig or its fragments (light or heavy chains). Approximately 25% of cases have
translocations that lead to overexpression of FGFR3, a receptor for fibroblast growth factor
implicated in control
of cell proliferation.

Small lymphocytic lymphoma (choice E), a B cell neoplasm, is equivalent to chronic lymphocytic
leukemia.
Neoplastic cells resemble a normal subset of circulating mature B cells that express CD5. Various
types of
chromosomal abnormalities have been found in small lymphocytic lymphoma/chronic lymphocytic
leukemia, but
none affecting the bcl-2 locus.

A 10-year-old boy with history of epilepsy and mental retardation is brought to a specialty clinic for
evaluation.
Physical examination is remarkable for several ovoid hypopigmented areas on the trunk and large
numbers of red
and yellow papules on the face, particularly near the mouth. Biopsy of the papules demonstrates
angiofibromata.
This patient is most likely to have which of the following central nervous system pathologies?

A. Acoustic neuromas

B. Capillary hemangioblastomas

C. Herniation of cerebellar tonsils into foramen magnum

D. Large cortical hamartomas

E. Leptomeningeal angiomatosis

Explanation:
6

The correct answer is D. The disease is tuberous sclerosis. The facial angiofibromata are also called
adenoma
sebaceum, and the hypopigmented patches on the trunk are called ash-leaf spots. This disease is one
of the
neurocutaneous disorders called phacomatoses. Tuberous sclerosis is inherited as an autosomal
dominant
trait, and epilepsy and mental retardation are commonly seen in this disorder. Large, firm, white
hamartomatous
nodules (tubers) are seen in the cortex and in subependymal sites. The tubers consist of aberrantly
arranged
neurons and/or glia. Patients may also have pancreatic cysts, renal angiomyolipomas, and cardiac
rhabdomyomas. Rarely, an astrocytoma will arise in a tuber.

Acoustic neuromas (choice A) are a feature of neurofibromatosis Type II.

Capillary hemangioblastomas (choice B) are a feature of Von Hippel-Lindau syndrome.

Herniation of cerebellar tonsils (choice C) is a feature of Arnold-Chiari malformation.

Leptomeningeal angiomatosis (choice E) is a feature of Sturge-Weber disease.

A 30-year-old African American woman has a chest x-ray that shows hilar lymphadenopathy and diffuse
abnormalities of the lung parenchyma. Biopsy reveals non-caseating granulomas. Acid fast, silver, and
PAS
stains on the granulomas are negative. Which of the following is the most likely diagnosis?

A. Coccidioidomycosis

B. Histoplasmosis

C. Leprosy

D. Sarcoidosis

E. Tuberculosis

Explanation:

The correct answer is D. Sarcoidosis is a common granulomatous disease, the etiology of which
remains
unclear. It is a diagnosis of exclusion, and mycobacterial (acid fast) and fungal (silver stain, PAS) are
used to
rule out infectious etiologies. The granulomas are characteristically small, non-caseating (hard)
granulomas that
may contain giant cells. Sarcoidosis has an initial predilection for the lungs and the hilar lymph nodes,
but with
time may spread to involve many organs. In the United States, sarcoidosis is much more common in
people of
African ancestry. In Europe, it mainly affects whites. There is a slight female preponderance. Patients
can have
7

a progressive course or experience exacerbations and remissions. Some patients recover with little
residual
damage; those who die typically do so from pulmonary fibrosis and cor pulmonale.

The causative agent of coccidioidomycosis (choice A), Coccidioides immitis, or the causative agent
of
histoplasmosis (choice B), Histoplasma capsulatum, would stain with PAS and silver stain.

The causative agent of leprosy (choice C), Mycobacterium leprae, or the causative agent of
tuberculosis
(choice E), Mycobacterium tuberculosis, would stain with acid fast stain.

A patient presents with a severe form of atopic asthma. Which of the following changes would most likely
be
found in this patient's blood?

A. Basophilic leukocytosis

B. Eosinophilic leukocytosis

C. Lymphocytosis

D. Monocytosis

E. Neutrophilic leukocytosis

Explanation:

The correct answer is B. An increased number of eosinophils (AKA eosinophilia) occurs in


association with
several conditions, the most frequent of which are immune-mediated diseases (e.g., asthma, hay fever,
and
pemphigus vulgaris) and parasitic infestations. This is due to an absolute increase in the number of
circulating
eosinophils, brought about by IL-5, which stimulates differentiation of eosinophilic precursor cells in
the bone
marrow.

Basophilic leukocytosis (choice A) is a rare event that is sometimes observed in association with
chronic
myelogenous leukemia. It is not seen in patients with asthma.

Lymphocytosis (choice C) may result from a vast array of conditions, but it is not typical of asthma or
other
allergic diseases. Lymphocytosis may develop in response to a number of infections (e.g., brucellosis,
whooping cough, hepatitis, infectious mononucleosis, and tuberculosis) or manifest as part of chronic
lymphocytic leukemia.

Monocytosis (choice D) refers to an increase in number of monocytes, which are circulating


macrophages.
Chronic infections (e.g., tuberculosis, rickettsiosis, and malaria) and chronic inflammatory conditions
(e.g.,
8

collagen vascular diseases and inflammatory bowel disease) are the most common underlying causes.

Neutrophilic leukocytosis (choice E) is a typical systemic reaction to acute and chronic infections,
especially
those due to bacteria. The increase in neutrophil number is mediated by IL-1 and TNF, which induce
a rapid
release of neutrophils from the bone marrow in acute infections and stimulate proliferation of bone
marrow
precursors in chronic infections.

A patient with long term severe hypertension develops progressive dementia. CT scan of the head
demonstrates
a diffuse loss of deep hemispheric white matter. Which of the following terms best describes the
pathological
process that is occurring?

A. Anemic infarcts

B. Hemorrhagic infarcts

C. Hypertensive encephalopathy

D. Lacunae

E. Subcortical leukoencephalopathy

Explanation:

The correct answer is E. This patient has subcortical leukoencephalopathy (Binswanger's disease),
which is one
of the neurologic syndromes associated with hypertension. It is uncommon, but obviously
devastating. The
histologic findings are diffuse, irregular loss of axons and myelin accompanied by widespread gliosis.
Small
infarcts may be seen in the frontal lobes. The pathologic mechanism may be damage caused by severe
arteriolosclerosis. None of the other choices would produce diffuse subcortical white matter
involvement.

Anemic infarcts (choice A) can be seen in hypertensive patients as a consequence of atherosclerotic


thromboembolic events.

Hemorrhagic infarcts (choice B) can also be seen in hypertensive patients as a consequence of


atherosclerotic
thromboembolic events followed by reperfusion. They tend to occur in gray matter or at the gray-
white junction.

Hypertensive encephalopathy (choice C) is an acute generalized dysfunction of the brain that can
occur in
malignant hypertension or other very severe hypertensive processes. The primary pathology is seen in
cerebral
vessels, although cerebral edema may be present.
9

Lacunae (choice D) are small necrotic foci in deep gray matter (especially basal ganglia and
thalamus) seen in
some hypertensive patients.

What percentage of Down's syndrome patients also have congenital cardiovascular disease?

A. 0.1%

B. 3%

C. 20%

D. 50%

E. 90%

Explanation:

The correct answer is C. This fact is worth remembering: one-fifth of Down's syndrome patients have
congenital cardiovascular disease, most commonly an ostium primum type of ASD and/or a
ventricular septal
defect. This is a sufficiently high incidence to justify at least one echocardiogram in each of these
children's
lives. Affected children are also particularly vulnerable to seizures, and as adults may develop an
Alzheimer-like dementia in their mid 40s.

A 50-year-old man presents with renal colic and an intravenous pyelogram demonstrates "clumps" of
contrast
medium limited to the medulla. Multiple small stones are also seen. Blood chemistries are all within
normal limits.
What is the most likely explanation for these findings?

A. Adult polycystic renal disease

B. Horseshoe kidney

C. Infantile polycystic renal disease

D. Medullary sponge kidney

E. Renal dysplasia

Explanation:

The correct answer is D. This is a classic presentation for medullary sponge kidney, which has a male
predominance and typically presents at 40-60 years of age. Histologically, medullary sponge kidney
shows
10

multiple small cysts lined by columnar or cuboidal epithelium localized to the medullary collecting
tubules. The
cysts can contain laminated concretions of calcium phosphates. Renal failure is rare in patients with
medullary
sponge kidney and the pathogenesis for the lesion has not been clarified. The rare uremic medullary
sponge
kidney is distinguished from the more common form by occurrence in 20-30 year-olds, salt-losing
nephropathy,
and progression to renal failure.

The kidney of adult polycystic disease (choice A) is massively enlarged and filled throughout with
round cysts
of varying sizes. Adult polycystic kidney disease often presents with hypertension rather than renal
failure or
stones.

Horseshoe kidney (choice B) involves fusion of the upper or lower (most common) pole of the
kidney. It is fairly
common (as high as 1:500), and is typically an incidental finding at autopsy.

Infantile polycystic kidney (choice C) produces a small kidney with round medullary cysts and
"radiating" linear
cortical cysts.

Renal dysplasia (choice E) can also cause cystic change in a kidney, but typically involves only the
medulla
and cortex of part of the kidney.

A 75-year-old female presents to the doctor with a chief complaint of vaginal spotting. She has been
post-menopausal for 25 years and does not take hormones. An ultrasound is performed, and shows a
mass in
the uterine fundus. A hysterectomy is performed, and pathologic examination of the removed uterus
reveals a
malignant tumor of the endometrial glands and stroma. Which of the following is the most likely
diagnosis?

A. Endolymphatic stromal myosis

B. Endometrial carcinoma

C. Endometrial stromal sarcoma

D. Leiomyosarcoma

E. Malignant mixed müllerian tumor

Explanation:

The correct answer is E. Malignant mixed müllerian tumor is a tumor with 2 components, stromal and
epithelial
(endometrial glands), both of which are malignant. This is a rare and highly aggressive tumor that has
a 25%
11

5-year survival rate. It usually affects older patients and presents with post-menopausal bleeding. The
stromal
component can contain metaplastic components such as cartilage and bone. Interestingly, usually only
the
epithelial component metastasizes.

Endolymphatic stromal myosis (choice A) is a type of endometrial stromal tumor of intermediate


malignancy. It
appears as small pieces of stroma between myometrial bundles that infiltrate lymph channels. Patients
may
have pain or bleeding, or may be asymptomatic. Recurrences happen late in the course of the disease
(years)
in half of patients and metastasis occurs in 15%. There is no epithelial component, so this is an
incorrect
choice.

Endometrial carcinoma (choice B) is a malignancy of the epithelial glandular component of the


endometrium.
Abnormal bleeding is the usual presentation. High estrogen states cause this tissue to proliferate.
There is no
stromal component of this tumor, so this is an incorrect choice.

Endometrial stromal sarcoma (choice C) is a true sarcoma arising from the endometrial stroma that
infiltrates
the myometrium and invades vessels. There is no epithelial component.

Leiomyosarcoma (choice D) is a true sarcoma arising from the uterine smooth muscle. It commonly
has satellite
lesions within the uterus. Leiomyosarcomas usually recur after removal; survival is greater with well
differentiated lesions. Poorly differentiated lesions have a 10 to 15% 5-year survival rate. Distant
metastasis is
via blood vessels. There is no epithelial component.

An 80-year-old man has low back pain. An x-ray of the lower back and pelvis shows sclerotic changes in
the lower
vertebrae and in focal areas throughout the pelvis. The radiologist's report states that the sclerotic
changes may
represent osteoarthritis; however, metastatic prostate cancer cannot be excluded. Which of the
following is most
cost-effective in the initial work-up of this patient?

A. Bone marrow aspirate and biopsy

B. Digital rectal exam

C. Prostate-specific antigen

D. Radionuclide bone scan

E. Serum alkaline phosphatase

Explanation:
12

The correct answer is B. Osteoarthritis is the most common rheumatologic disease, the prevalence of
which
increases with age. It primarily involves weight-bearing joints, hence its distribution in the lower
vertebrae, pelvic
bones, and proximal femur. Sclerotic bone, representing reactive bone formation, develops as a
reaction to
injury and is responsible for the slightly elevated serum alkaline phosphatase levels that normally
occur in much
of the elderly population. If prostate cancer with osteoblastic (bone-forming) metastases to the
vertebral column
and pelvis were present in this patient (stage D disease), a digital rectal exam would be the most cost-
effective
initial step in the work-up. With advanced prostate cancer, the gland would very likely be enlarged
and hard
("stony").

A bone marrow aspirate and biopsy (choice A) is not usually part of the normal work-up of possible
metastatic
prostate cancer and has no place in the evaluation of osteoarthritis.

A prostate-specific antigen (PSA; choice C) level should be ordered in this patient, but not as the
initial step in
the work-up, since it does not distinguish hyperplasia from cancer and is fairly expensive. In known
cases of
prostate cancer, the PSA is a measure of tumor burden and is used to monitor recurrences when
following
patients who have been treated for prostate cancer.

A radionuclide bone scan (choice D) is commonly used to rule out metastasis in patients with prostate
cancer. It
is expensive and is not used as a screening test for prostate cancer.

The serum alkaline phosphatase (choice E) is typically elevated in metastatic prostate cancer due to
osteoblastic activity in the metastatic foci. However, an elevated serum alkaline phosphatase is non-
specific,
since it may be slightly increased in osteoarthritis (reactive bone formation) as well as in liver disease.

A 40-year-old woman presents to the emergency department with severe abdominal pain localized to the
right
upper quadrant. A urine sample is taken for rapid dipstick reagent strip analysis. A positive result for
which of the
following substances would most strongly suggest gallstone disease as a possible cause of her
abdominal pain?

A. Bilirubin

B. Glucose

C. Nitrite

D. Protein
13

E. Urobilinogen

Explanation:

The correct answer is A. A small gallstone passing into the common bile duct can cause obstructive
jaundice
with conjugated hyperbilirubinemia. The conjugated bilirubin will spill into the urine, causing the
bilirubin square
on the reagent strip to react. This strip may also react with other causes of intrahepatic or extrahepatic
obstructive jaundice, so it is not completely specific. It is worth becoming very familiar with the
strengths and
weaknesses of reagent strip technology, however, since this information may be available hours
before serum
chemistry values are reported.

High glucose (choice B) in urine suggests diabetes mellitus.

High nitrite (choice C) in urine suggests urinary tract infection.

High protein (choice D) in urine suggests renal disease or myeloma.

Urobilinogen levels (choice E) in obstructive jaundice can be normal, raised, or lowered;


consequently, they are
not diagnostically helpful in the setting described in the question stem unless urinary bilirubin is
negative.

A 60-year-old man presents to his physician after a routine screening test indicates hyperlipidemia.
Physical
examination reveals raised, irregular, yellow papules in the skin of the soft tissues below the eyes.
Biopsy of
these lesions would most likely show which of the following?

A. Benign nevus cells

B. Malignant nevus cells

C. Microscopic blisters

D. Munro microabscesses

E. Multinucleated giant cells

Explanation:

The correct answer is E. The lesions are xanthomas, which are tumor-like dermal collections of
foamy
histiocytes containing cholesterol and lipids. The lesions may also contain multinucleated giant cells
(Touton
giant cells) with clustered nuclei and foamy cytoplasm. Xanthomas may be idiopathic or may be
related to
hyperlipidemia or lymphoproliferative malignancies (e.g., leukemias and lymphomas).
14

Nevus cells are a type of melanocyte. Benign nevus cells (choice A) are a feature of the common
nevus, or
mole. Malignant nevus cells (choice B) are a feature of malignant melanoma.

Microscopic blisters (choice C) at the dermal/epidermal junction are a feature of dermatitis


herpetiformis.

Munro microabscesses (choice D), small collections of neutrophils in the cornified epidermis, occur
in psoriasis.

A 17-year-old male develops a painless, firm mass beneath the nipple of his left breast. The mass is mobile,
and
no fluid can be expressed from the breast. The right breast is normal to examination. Which of the
following
conditions does this mass most likely represent?

A. Fibrocystic changes

B. Gynecomastia

C. Intraductal papilloma

D. Invasive duct carcinoma

E. Invasive lobular carcinoma

Explanation:

The correct answer is B. The most common breast mass in men, especially under 25 years of age, is
gynecomastia-a benign proliferation of ductal and stromal elements of the breast. It is generally an
idiopathic
condition, probably related to pubertal hormonal changes.

Fibrocystic changes (choice A) in the breast, which reflect physiological responses in the breast to
cyclical
levels of sex hormones, are not observed in men. Fibrocystic changes are most common in the late
reproductive years and include fibrosis, cyst formation, and a variety of epithelial changes such as
hyperplasia
and apocrine metaplasia.

Intraductal papillomas (choice C) are benign neoplasms commonly evolving in the major lactiferous
ducts
beneath the nipple. They most commonly present with a bloody nipple discharge and are rare in men.

Carcinoma of the male breast (choices D and E) is rare, and almost always develops in the breasts of
elderly
men. Male breast carcinomas have a somewhat worse prognosis than their female counterpart.
Grossly and
microscopically they resemble ductal carcinoma in the female.
15

Physical examination of a 45-year-old man who looks much older than his stated age demonstrates thin
arms and
legs, a swollen abdomen, red tongue, dry, thin, and slightly yellow skin, gynecomastia, testicular
atrophy, multiple
spider angiomas, tremor, yellow discoloration of sclera, and short-term memory loss. Which of the
following
conditions is most strongly suggested by these findings?

A. Bronchogenic carcinoma

B. Colon carcinoma

C. Congestive heart failure

D. Glomerulonephritis

E. Hepatic cirrhosis

Explanation:

The correct answer is E. The physical examination is typical for an advanced alcoholic with hepatic
cirrhosis. It is
important to recognize these symptoms, as these patients are notorious for "underestimating" and
even denying
their alcohol use.

Bronchogenic carcinoma (choice A) typically presents with cough and/or respiratory changes, but can
present
with mass effects in the chest or involvement of mediastinal nerves or vessels.

Colon cancer (choice B) typically presents with changes in the stool or bowel habits.

Congestive heart failure (choice C) is typically heralded by shortness of breath or peripheral edema,
or both.

Glomerulonephritis (choice D) typically presents with changes in quality or quantity of urine, and,
possibly, fluid
retention.

A 54-year-old white male presents with gradual onset of mild dementia, ataxic gait, and startle myoclonus.
An MRI
scan is normal, and an examination of his cerebrospinal fluid reveals no abnormalities, but the
patient's EEG is
remarkable for recurrent bursts of high-voltage slow waves. Over the next 6 months, the patient's
dementia
rapidly worsens, accompanied by general hypertonicity and profound dysarthria. The patient dies
shortly
thereafter. Which of the following is the mostly likely neuropathological finding on autopsy?

A. Cerebellar hyperplasia
16

B. Diffuse spongiform change

C. Multiple lacunar infarcts

D. Negri bodies

E. Neurofibrillary tangles

Explanation:

The correct answer is B. The rapidly progressive dementia in this case is characteristic of Creutzfeldt-
Jakob
disease (CJD). The dementia is usually accompanied by motor dysfunction and abnormal EEG
activity, as
described in the question stem. The pathological hallmark of this disease is spongiform change in the
gray
matter. Death usually occurs within 6-12 months of disease onset.

At autopsy, the cerebellum in CJD appears atrophic, not hyperplastic (choice A).

Multiple lacunar infarcts (choice C) are seen with vascular dementia, and patients typically present
with focal
neurological signs. Additionally, vascular dementia typically presents with a more gradual decline in
cognitive
function, measured in years rather than months.

Negri bodies (choice D) are pathognomic for rabies, which does not cause dementia.

Neurofibrillary tangles (choice E) are often seen in patients with Alzheimer's disease. Alzheimer's
disease
primarily affects the higher order association cortex; motor dysfunction is not generally observed.
Additionally,
in Alzheimer's disease, there is a more gradual decline in cognitive function, measured in years rather
than
months.

A 38-year-old woman is in her first pregnancy, which has been uneventful until the 34th week, when she
develops swelling of feet and hands. An obstetric check-up reveals that she also has hypertension and
proteinuria. Laboratory analysis shows elevated aspartate aminotransferase (AST) and alanine
aminotransferase
(ALT) and slightly decreased platelets. The initial event in the pathogenesis of her condition is
thought to be
which of the following?

A. Chorioamnionitis

B. Disseminated intravascular coagulation

C. Maternal hypertension

D. Maternal renal ischemia


17

E. Placental ischemia

Explanation:

The correct answer is E. This patient's condition is a classic third trimester complication referred to as
toxemia
of pregnancy, or preeclampsia. It occurs in 6% of all pregnancies but is more frequent in primiparas.
Although
the pathogenesis is still unclear, the first event appears to be placental ischemia, probably due to
abnormalities
in the trophoblast and alterations in the maturation of placental vessels. The trophoblast of invading
placental
vessels fails to acquire the characteristics of normal endothelial cells, with subsequent alterations in
blood flow.
Placental ischemia then triggers the release of thromboplastic substances, increases renin synthesis,
and
reduces prostaglandin E levels.

Chorioamnionitis (choice A) is an infection of chorioamnionic membranes due to bacteria that ascend


through
the vaginal canal. This is an important cause of spontaneous abortion in the second and third
trimester, but it
plays no role in the pathogenesis of toxemia of pregnancy.

The release of thromboplastic substances may cause disseminated intravascular coagulation (DIC)
(choice B).

Increased renin and reduced prostaglandin E mediate increased sensitivity to angiotensin, leading to
maternal
hypertension (choice C).

Ischemic damage to maternal organs, including brain, liver, and kidneys (choice D), results from
thrombotic
occlusion of arterioles and capillaries as a consequence of DIC.

Overall, the clinical picture of toxemia is due to DIC-mediated ischemic damage to brain (changes in
mental
status and convulsions), liver (elevated liver enzymes), and kidneys (proteinuria, leading to peripheral
edema).
A manifestation of toxemia is HELLP syndrome, which stands for hemolysis, elevated liver enzymes,
and low
platelets.

Molecular studies on an abdominal lymph node containing lymphoma demonstrate t(2;8)(p12;q24)


translocation.
This is most compatible with which of the following diseases?

A. Burkitt's lymphoma

B. Mantle cell lymphoma


18

C. Multiple myeloma

D. Small cell lymphoma

E. Small cleaved cell lymphoma

Explanation:

The correct answer is A. Burkitt's lymphoma is actually associated with three translocations. The
common
variant t(8;14)(q24;q32), involving the oncogene myc on chromosome 8, and the heavy
immunoglobulin chain
on chromosome 14. The other two variants are: t(8;22)(q24;q11), involving myc and the lambda light
chain
immunoglobulin site, and t(2;8)(p12;q24), involving the kappa light chain and myc.

Mantle cell lymphoma (choice B), multiple myeloma (choice C), and small (not cleaved) cell
lymphoma (choice
D) are associated with the t(11;14)(q13;q32) translocation involving bcl -1 and the heavy chain site.

Small cleaved cell lymphoma (choice E) is associated with t(14;18)(q 32;q21), involving the
immunoglobulin
chain site and bcl-2.

A newborn infant is noted to have numerous, light brown macules dispersed across her skin. The
significance of
this feature is due to its strong association with the development of which of the following tumors?

A. Basal cell carcinoma

B. Neuroblastoma

C. Neurofibroma

D. Retinoblastoma

E. Wilms' tumor

Explanation:

The correct answer is C. Congenital "cafe au lait" spots are present in more than 90% of patients with
neurofibromatosis. This autosomal dominant disorder is characterized by multiple neural tumors,
especially
neurofibromas, pigmented hamartomas of the iris (Lisch nodules), and cafe au lait spots, which
usually occur
over nerve trunks. Although the majority of neurofibromas in this disease are benign, the tumors can
be quite
disfiguring and psychologically damaging.

Basal cell carcinoma (BCC; choice A) is a common, slow-growing tumor of sun-exposed skin. It
develops in
19

adulthood after years of chronic sun damage. Although malignant melanoma may arise from large
congenital
nevi, BCC does not.

Neuroblastoma (choice B) is a common childhood tumor that arises anywhere along the sympathetic
chain, and
most commonly in the adrenal medulla. The tumor usually presents as an abdominal mass and is not
associated with skin findings.

The ocular neoplasm associated with neurofibromatosis is the Lisch nodule, not the retinoblastoma
(choice D).
Retinoblastoma is a neuroepithelial tumor usually identified by funduscopic examination of a child
with visual
changes.

Wilms' tumor (choice E) is a neoplasm of primitive renal blastema that may be associated with
congenital
malformations of visceral organs, notably the adrenals and gonads, but not skin. It also presents as an
abdominal mass, and is highly survivable with modern treatment modalities.

A neonate develops bile-stained vomiting and progressive abdominal distention, and does not pass
meconium
over the first two days of life. The anus is patent, and the bowel loops are palpable. Plain radiograph
shows
bubbly meconium in the right lower quadrant. No localized areas of constriction or other
abnormalities are noted.
Which of the following is most likely etiologically related to this infant's condition?

A. Cystic fibrosis

B. Hirschsprung's disease

C. Meckel's diverticulum

D. Omphalocele

E. Polycystic kidney disease

Explanation:

The correct answer is A. The baby has meconium ileus, which is a manifestation of cystic fibrosis due
to the
abnormally viscid pancreatic secretions which "get stuck" in the small bowel. Meconium ileus can
cause gut
perforation with peritonitis and intraperitoneal calcifications (that may be visible on plain film).
Meconium ileus
complicated by intestinal perforation or formation of fistulas to the bladder or vagina must be treated
surgically.
Medical treatments for uncomplicated meconium ileus are now available, which use enemas,
mucolytic agents,
or pancreatic enzymes.
20

Hirschsprung's disease (choice B) is a cause of congenital constipation related to absence of ganglion


cells in a
segment of bowel. The aganglionic bowel segment is narrowed because the lack of peristalsis keeps
stool from
moving into the segment. The distal rectum is always involved, and the lesion may extend proximally
as far as
the small intestine. The bowel proximal to the lesion is usually dilated.

A Meckel's diverticulum (choice C) can form due to the persistence of the vitelline duct, which
connects the
developing gut to the yolk sac. They are classically located in the distal ileum within 30 cm of the
ileocecal
valve, and may contain ectopic pancreatic tissue or gastric mucosa.

Omphalocele (choice D) is characterized by herniation of abdominal viscera through the abdominal


wall near
the umbilicus.

In polycystic kidney disease (choice E), cysts of the liver and pancreas may occur; meconium ileus is
not
associated with this disorder.

A 69-year-old woman is brought to the emergency room after falling off a step stool and fracturing her hip.
Her
past medical history is significant for several bouts of pneumonia during the past year. Laboratory
results indicate
a normal white blood cell count, but platelets are decreased, and the erythrocyte sedimentation rate
(ESR) is
elevated. X-rays reveal multiple lytic bone lesions. Serum electrophoresis demonstrates an M-protein
spike.
Which of the following is the most likely diagnosis?

A. Chronic lymphocytic leukemia

B. Monoclonal gammopathy of uncertain significance

C. Multiple myeloma (plasma cell myeloma)

D. Plasmacytoma

E. Waldenström's macroglobulinemia

Explanation:

The correct answer is C. The patient is suffering from multiple myeloma, a neoplastic proliferation of
plasma
cells (or their precursors) found within the bone marrow. These malignant cells are responsible for the
production of excessive amounts of immunoglobulin (usually IgG or IgA), producing an M- protein
spike, and
increasing the ESR. The decreased platelet count probably reflects infiltration of the bone marrow by
myeloma
cells. It is not unusual for patients with malignant myeloma to have recurring bacterial infections,
particularly
21

pneumococcal pneumonia, because the overall production of normal immunoglobulins of all isotypes
is
decreased. Lytic bone lesions due to infiltration by myeloma cells may lead to pathological fractures.
All of the
other disorders listed can be associated with an M-protein spike, but this is a nonspecific finding.

Chronic lymphocytic leukemia (CLL; choice A) is ruled out by the normal white count, since this
type of leukemia
is generally associated with an absolute lymphocytosis.

Patients with monoclonal gammopathy of undetermined significance (choice B) are generally


asymptomatic,
although they have a predisposition for subsequently developing myeloma, lymphoma, amyloidosis,
or
Waldenström's macroglobulinemia.

Plasmacytoma (choice D) (solitary myeloma) is a rare, isolated plasma cell neoplasm in bone or soft
tissues. If
the primary cancer is in bone, it is likely to disseminate; extraosseous tumors tend to remain localized.
Since this
patient had multiple lytic bone lesions, multiple myeloma is the correct diagnosis.

Waldenström's macroglobulinemia (choice E) is a disorder involving neoplasms of lymphocytoid


plasma cells
that produce monoclonal IgM. Hypergammaglobulinemia produces hyperviscosity of the blood
which, along with
infiltration by tumor produce the characteristic signs and symptoms. Weakness, weight loss, bone
pain,
hepatosplenomegaly, and lymphadenopathy occur commonly.

A 55-year-old woman complains to her physician that the skin of her armpits and groin "keeps getting
darker and
darker." Physical examination demonstrates velvety brown and warty skin in the axilla and groin.
Biopsy of these
lesions shows a variably hyperplastic epidermis with many sharp peaks and valleys. Aside from
cosmetic
considerations, which of the following is the primary medical significance of these lesions?

A. They may be a sign of immunosuppression

B. They may be a sign of visceral carcinoma

C. They may be easily superinfected

D. They may be malignant

E. They may be premalignant

Explanation:
22

The correct answer is B. The lesions are acanthosis nigricans, which looks somewhat like a mole or
wart, but is
actually due to epidermal hyperplasia. Acanthosis nigricans can be seen in obesity, diabetes, and in
patients
with underlying cancers, often adenocarcinomas of the chest or abdomen.

The lesions are not characteristic of immunosuppression (choice A) and are not easily superinfected
(choice C).
They are also neither malignant (choice D) nor premalignant (choice E).

A 47-year-old woman undergoes endometrial biopsy because she has had repeated episodes of irregular
spotting between periods lately. The biopsy shows strips of endometrium bearing long, narrow, coiled
glands
lined by a single layer of columnar epithelium showing regular, uniform, small nuclei and clear apical
vesicles.
Which of the following is the most likely diagnosis?

A. Atypical hyperplasia

B. Complex hyperplasia without atypia

C. Proliferative endometrium

D. Secretory endometrium

E. Simple hyperplasia

Explanation:

The correct answer is D. Endometrial biopsies are often performed to evaluate patients with menstrual
bleeding
abnormalities, particularly in perimenopausal or postmenopausal patients. While the biopsies are done
to rule
out hyperplasias or cancer, most of the specimens actually show only proliferative or secretory (as in
this
patient) endometrium. Secretory endometrium has the features noted in the question stem.

Atypical hyperplasia (choice A) is characterized by complex glands lined by cells showing features of
atypia,
such as cytomegaly, increased nuclear cytoplasmic ratio, prominent nucleoli, and increased mitotic
index.

Complex hyperplasia without atypia (choice B) is characterized by complex, branching glands


without cellular
atypia.

Proliferative endometrium (choice C) is characterized by smaller, noncoiled, glands lined with


columnar
epithelium without apical (secretory) vesicles.

Simple (cystic) hyperplasia (choice E) is characterized by cystically dilated glands without cellular
atypia.
23

Which of the following conditions is the most frequent cause of spontaneous abortion in the first trimester
of
pregnancy?

A. Abruptio placentae

B. Chorioamnionitis

C. Chromosomal abnormalities

D. Placenta previa

E. Trauma

Explanation:

The correct answer is C. At least 10% to 15% of normally fertilized and implanted ova are lost in the
first
trimester of pregnancy because of spontaneous abortion. Studies using immunoassay of human
chorionic
gonadotropin (hCG) for early diagnosis of pregnancy suggest that the percentage of fertilized ova lost
in the
first trimester might be even higher. The great majority of these cases are attributable to chromosomal
abnormalities. Chromosomal studies are not routinely performed in such cases, but they are
recommended
when a malformed fetus has been identified or when habitual or recurrent abortions occur.

Abruptio placentae (choice A), a complication of the third trimester, occurs when the placenta
detaches
prematurely from its implantation site. Retroplacental hemorrhage develops within the space between
placenta
and uterine wall, leading to interruption or severe reduction in the blood supply to the fetus.

Chorioamnionitis (choice B), a complication of the second and third trimesters, results from
ascending infections
through the vaginal canal. Infection of chorioamnionic membranes may lead to premature rupture of
membranes and abortion or premature labor.

Placenta previa (choice D) is a placenta implanted in the lower segment of the uterus. When dilatation
of this
segment begins in late pregnancy, a placenta previa may cause severe bleeding and lead to premature
labor.

Surprisingly, trauma (choice E) is a rare cause of spontaneous abortion.

A 65-year-old woman has a long-standing dementing disorder characterized by deterioration in personality,


neglect of personal hygiene, impaired judgment, and disinhibited behavior. MRI demonstrates severe
cortical
24

atrophy limited to the frontal lobes and anterior two thirds of the temporal lobes, while the remaining
cortex is
preserved. No evidence of recent or remote infarcts is found. Which of the following diagnoses is
most consistent
with these pathologic and clinical features?

A. Alzheimer disease

B. Creutzfeldt-Jacob disease

C. Dementia with Lewy bodies

D. Frontotemporal dementia

E. Vascular dementia

Explanation:

The correct answer is D. Not all dementing disorders manifest with the same clinical features.
Although there is
considerable overlap in clinical symptomatology among different types of dementias, making clinical
diagnosis
somewhat problematic, there are classic presentations that allow identification of a specific form of
dementia
with a high degree of confidence. In this case, the patient has symptoms due to frontal lobe damage,
eg,
disinhibition, impaired judgment, and personality changes. Furthermore, MRI demonstrates a specific
pattern of
cortical atrophy, restricted to the frontal lobes and anterior portion of the temporal lobes. This
combination
points toward a group of dementias called frontotemporal dementia, the most frequent form of which
is Pick
disease. Other forms of frontotemporal dementia are very infrequent. Remember: frontal symptoms in
conjunction with frontotemporal atrophy = frontotemporal dementia/Pick disease.

Alzheimer disease (choice A) is the most frequent form of dementia in industrialized countries.
Although
symptoms due to frontal damage may be present in Alzheimer disease, they are usually associated
with a more
generalized impairment of higher neurologic functions, eg, language, memory, and learned
movements. In
addition, cortical atrophy in Alzheimer disease is widespread and not limited to the frontal and
anterior temporal
lobes.

Creutzfeldt-Jacob disease (choice B) represents the prototype of prion diseases. Cortical atrophy is
not a
prominent feature of Creutzfeldt-Jacob disease, which manifests with personality changes, memory
loss, and
seizures, leading to death after a rapid clinical course (a few months to 1 year).

Dementia with Lewy bodies, also known as diffuse Lewy body disease (choice C), is one of the most
common
25

forms of dementia in Western countries, possibly more common than vascular dementia. It is
characterized by
widespread formation of Lewy bodies in the substantia nigra, limbic cortex, and subcortical nuclei,
such as the
basal nucleus of Meynert. Extrapyramidal symptoms similar to Parkinson disease manifest in this
form of
dementia as a result of degeneration of dopaminergic pathways.

Vascular dementia (choice E) is an umbrella term encompassing dementing conditions that arise from
pathology
of large or small cerebral vessels. It manifests with memory loss associated with focal neurologic
symptoms
depending on the location of damage. MRI would identify old or recent infarcts, as well as white
matter disease.
Conditions associated with vascular dementia include the following:
Multi-infarct dementia, which is caused by multiple, scattered brain infarcts secondary to
atherosclerosis of
large arteries of the circle of Willis and/or carotid arteries.
Binswanger disease, which involves rarefaction of cerebral white matter and is caused by
hypertension-related
arteriolosclerosis.
Lacunar infarcts, which consist of small (< 1 cm) infarcts in the striatum and thalamus; this condition
is related
to arteriolosclerosis.

Biopsy of a reasonably well-demarcated mass of the nasopharynx demonstrates a plasma cell proliferation.
Serum electrophoresis shows a small monoclonal IgG spike. Bone marrow evaluation fails to
demonstrate plasma
cell proliferation and no lesions are seen on extensive skeletal x-rays. Which of the following is the
most likely
diagnosis?

A. Heavy chain disease

B. Monoclonal gammopathy of undetermined significance

C. Multiple myeloma

D. Plasmacytoma

E. Waldenström's macroglobulinemia

Explanation:

The correct answer is D. Plasmacytoma (solitary myeloma) involving soft tissue (lungs, nasopharynx,
nasal
sinuses) is a plasma cell proliferation resembling multiple myeloma but without significant metastatic
potential. In
contrast, some plasmocytomas involving bone eventually (up to 10 to 20 years) develop into frank
multiple
myeloma.
26

Heavy chain diseases (choice A) constitute a group of rare lymphoplasmacytic malignancies in which
excessive
amounts of a defective heavy immunoglobulin chain are produced. They may take the form of gamma
heavy-chain disease (from IgG), alpha heavy-chain disease (from IgA) or mu heavy-chain disease
(from IgM);
malignant cells are usually present in marrow in all of these conditions.

Monoclonal gammopathy of undetermined significance (choice B) is a disease of elderly patients with


a
monoclonal spike on serum or urine electrophoresis, but no identifiable mass or bone marrow lesion;
20% of
these patients eventually develop one of the other diseases listed in the answer choices.

Multiple myeloma (choice C) is a malignancy derived from a single plasma cell clone with significant
metastatic
potential. Multiple lytic bone lesions are usually seen.

Waldenström's macroglobulinemia (choice E) is a malignancy of lymphoplasmacytic cells that secrete


IgM. In this
disorder, the bone marrow is diffusely rather than focally infiltrated by lymphocytes, plasma cells,
and hybrid
forms.

A 65-year-old woman presents with a 30-lb weight loss and malaise. Cancer is suspected. Which of the
following
is the correct list, starting with the most prevalent, of the three most common causes of cancer in
women?

A. Breast, lung, colon and rectum

B. Breast, uterus, lung

C. Colon and rectum, lung, ovary

D. Lung, breast, ovary

E. Ovary, uterus, lung

Explanation:

The correct answer is A. The correct female incidence sequence is breast (32%), lung (13%), and
colon and
rectum (13%). The two major causes of cancer death in women are lung (23%) and breast (18%). In
men, the
incidence sequence is prostate (32%), lung (16%), and colon and rectum (12%). The two leading
causes of
male cancer deaths are lung (33%) and prostate (13%).

The uterus and ovary (choices B, C, D, and E) are not among the three organs most frequently
affected by
cancer in women.
27

A 50-year-old woman presents with a 5-year history of headaches, generalized tonic-clonic seizures, and
bilateral
leg weakness. Skull films reveal hyperostosis of the calvarium. Biopsy of the responsible lesion shows
a whorling
pattern of the cells. Which of the following is the most likely diagnosis?

A. Arachnoid cyst

B. Glioblastoma multiforme

C. Meningioma

D. Metastatic breast cancer

E. Oligodendroglioma

Explanation:

The correct answer is C. The most likely diagnosis is an intracranial meningioma. Meningiomas are
slow-growing, benign tumors comprising 15% of intracranial tumors; they are most common in the
elderly. They
originate from either dura mater or arachnoid and are sharply demarcated from brain tissue.
Meningiomas often
incite an osteoblastic reaction in the overlying cranial bones. Microscopically, the meningioma cells
have a
tendency to encircle one another, forming whorls and psammoma bodies. Clinically, they present as
mass
lesions; seizures may occur. The superior parasagittal surface of the frontal lobes is a favorite site of
origin.
This can often produce leg weakness, since the leg motor fibers that pass down through the internal
capsule
originate in parasagittal cortical regions. Treatment of meningiomas is usually surgical.

Arachnoid cysts (choice A) are formed by splitting of the arachnoid membrane; most arachnoid cysts
arise near
the Sylvian fissure. They may present with mass effect, but would be unlikely to produce seizures,
prominent
focal signs, or reactive hyperostosis.

Glioblastoma multiforme (choice B) is an aggressive malignant astrocytoma that would likely have
killed the
patient long before 5 years had elapsed.

Metastatic breast cancer (choice D) would generally look different microscopically (the whorling cell
pattern is
characteristic of meningioma). It would be unlikely for metastatic cancer to cause a reaction in the
overlying
bone, or to be present long enough to cause symptoms for 5 years.

Oligodendrogliomas (choice E) are glial tumors that could produce the described clinical picture, but
usually do
not cause hyperostosis of the calvarium or exhibit the characteristic whorling cell pattern
microscopically.
28

When a histologic section is taken of an abscess, many of the observed neutrophils show a degenerative
change
in which the nucleus has undergone fragmentation. This process is known as

A. caseous necrosis

B. coagulative necrosis

C. karyolysis

D. karyorrhexis

E. pyknosis

Explanation:

The correct answer is D. Karyorrhexis refers to a pattern of nuclear degradation in which a pyknotic
or partially
pyknotic nucleus undergoes fragmentation followed by complete lysis. This pattern is common in the
neutrophils present in acute inflammation.

The type of necrosis seen in an abscess is liquefactive necrosis. Caseous necrosis (choice A) is seen in
tuberculosis and some other granulomatous diseases; coagulative necrosis (choice B) is seen
following
infarctions of many organs (other than the brain).

Karyolysis (choice C) is also a degenerative change affecting nuclei. In this case, however, it is seen
as a
decrease in nuclear basophilia, which is presumably the result of DNAse activity.

Pyknosis (choice E) is characterized by nuclear shrinking and basophilia, apparently as a result of


DNA
condensation.

A 52-year-old woman has recently undergone a breast resection for carcinoma. Based on the statistics for
breast
cancer incidence, which of the following types of carcinoma does this patient most likely have?

A. Colloid (mucinous)

B. Invasive ductal

C. Invasive lobular

D. Medullary

E. Metastatic bronchogenic

Explanation:
29

The correct answer is B. Invasive ductal carcinoma is the most likely candidate. Of the various types
of breast
adenocarcinoma, invasive ductal carcinoma is by far the most common variant, accounting for
approximately
75% to 80% of all invasive breast carcinomas. Invasive ductal carcinoma develops from epithelial
cells of the
terminal duct. Histologically, it is composed of small, glandular, ductlike structures, lined by variably
anaplastic
cells. The most common mode of presentation is a palpable mass in the breast. Its prognosis depends
mostly
on staging (spread of cancer) rather than grading (degree of differentiation).

The colloid (mucinous) variant (choice A) is relatively rare (about 1% to 2%) and occurs more
frequently in
older women. Histologically, this carcinoma is characterized by abundant mucin secretion. It is
associated with a
better prognosis than the ductal type.

Invasive lobular carcinoma (choice C) is the second most frequent histologic type of breast
adenocarcinoma,
accounting for approximately 10% of all cases. Its presumed cell of origin is the lobular cell. The
most typical
histologic characteristic is the presence of cancer cells lined up in orderly rows ("single-file").

Medullary carcinoma (choice D) tends to occur in younger women and is associated with a slightly
better
prognosis. Although a malignant tumor, medullary carcinoma is well circumscribed and surrounded
by a florid
lymphoplasmacytic reaction. The name is due to its soft consistency.

Metastatic cancer may involve the breast like any other organ. Bronchogenic carcinoma (choice E)
may also
spread to the breast by lymphatic route or by contiguity, but this would be less likely than primary
breast
cancer.

A 55-year-old man is brought to his physician's office with a 3-month history of progressive mental
deterioration
in the form of memory loss, mood changes, and errors in judgment. His gait is unsteady, and he
requires
assistance to prevent falling. He has no history of seizures, head trauma, or incontinence. Computed
tomography
(CT) scan and lumbar puncture are unremarkable. Physical examination reveals hypertonicity of all
extremities,
bilateral equivocal plantar response, ataxic gait, and myoclonic jerks in the lower extremities. What is
the
mechanism by which this infectious agent causes its pathology?

A. Amyloid deposition

B. Autoimmune destruction
30

C. Chronic inflammation

D. Embolization and infarction

E. Toxin production

Explanation:

The correct answer is A. This is the classic presentation of Creutzfeldt-Jacob disease (CJD). Although
the
pathogenesis is incompletely understood, these patients develop extracellular deposition of abnormal
fibrillar
proteins in the brain, ie, amyloid.

Autoimmune destruction (choice B) is not indicated because there is no immunologic response to the
deposition of these extracellular proteins; thus there is no chronic inflammation (choice C)

Although embolization and infarction (choice D) could complicate the presentation in the age group
typically
afflicted with CJD, these processes are not believed to have any direct role in this pathology.

No toxin is produced (choice E) to account for the presentation in CJD.

A patient consults a dermatologist about a skin lesion on her neck. Examination reveals a 1-cm diameter,
red,
scaly plaque with a rough texture and irregular margins. Biopsy demonstrates epidermal and dermal
cells with
large, pleomorphic, hyperchromatic nuclei. Which of the following conditions would most likely
predispose this
patient to the development of this lesion?

A. Actinic keratosis

B. Compound nevus

C. Dermal nevus

D. Junctional nevus

E. Melanoma

Explanation:

The correct answer is A. The lesion is a squamous cell carcinoma of the skin. Actinic keratosis, which
is a
hyperplastic lesion of sun-damaged skin, predisposes for squamous cell carcinoma. Another
predisposing
condition to remember is xeroderma pigmentosum, which predisposes for both squamous cell and
basal cell
carcinomas of skin.
31

A nevus is a mole, containing characteristic cells called nevocellular cells. If the nevocellular cells are
located at
the dermal-epidermal junction (junctional nevus, choice D), in the dermis (dermal nevus, choice C),
or both
(compound nevus, choice B), they do not predispose for squamous cell carcinomas of the skin.
Malignant
melanoma (choice E), however, can arise in pre-existing nevi.

A 24-year-old woman gives birth to an apparently normal infant. The neonate begins feeding well by the
second
day, then at ten days, suddenly develops gastrointestinal obstruction. Which of the following is the
most likely
cause of this presentation?

A. Adhesions

B. Congenital pyloric stenosis

C. Hirschsprung's disease

D. Intussusception

E. Volvulus

Explanation:

The correct answer is D. All of the conditions listed can cause gastrointestinal obstruction, but the
clinical
presentation is most suggestive of intussusception. In intussusception, there is telescoping of one
bowel
segment into another, more distal segment. The disorder is relatively common in infants and children
due to the
poor support offered by their thin mesentery. Intussusception produces intestinal obstruction, and it
may
produce bowel ischemia or infarction by trapping mesenteric vessels along with the affected segment.
In some
cases, the intussusception may be reduced by diagnostic barium enema.

Adhesions (choice A) can cause bowel obstruction following surgery or inflammatory bowel diseases.

Congenital pyloric stenosis (choice B) typically presents as projectile vomiting in a 3-4 week old
baby.

Hirschsprung's disease (choice C), caused by absence of ganglion cells in the distal bowel, is usually
diagnosed in the first few days of life when there is a failure to pass meconium.

Volvulus (choice E) is due to rotation of bowel segments. This is usually a disease of the elderly.

A middle-aged man with a long history of sexual activity slowly develops testicular enlargement. Needle
biopsy
32

demonstrates the presence of obliterative endarteritis with perivascular cuffing of lymphocytes and
plasma cells.
A diffuse interstitial inflammation with edema and prominent plasma cell infiltrate is also present.
Which of the
following is the most likely diagnosis?

A. Gonorrhea

B. Mumps

C. Nonspecific orchitis

D. Syphilis

E. Tuberculosis

Explanation:

The correct answer is D. This is one presentation of syphilis, which can involve the testis and
epididymis in both
the acquired and congenital types of the disease. Characteristically, the testis is usually involved
before the
epididymis. Microscopically, the testis may show either gumma formation or the findings described
in the
question stem.

Gonorrhea (choice A) usually causes an acute epididymitis and orchitis with prominent neutrophils.

Mumps orchitis (choice B) would usually be accompanied by parotitis.

Nonspecific orchitis (choice C) is characterized by prominent neutrophils.

Tuberculosis orchitis (choice E) is characterized by granulomas with acid-fast bacteria.

A 70-year-old woman dies in a nursing home after a heart attack. The time of onset of her clinical
symptomatology and the cause of death are uncertain; furthermore, the possibility of neglect is being
considered. Therefore, an autopsy investigation is arranged. The forensic pathologist discovers acute
thrombosis involving the posterior descending branch of the right coronary artery with resultant
myocardial
infarction (MI) in the posterior third of the interventricular septum. Histologically, there is
coagulation necrosis
with associated abundant neutrophilic infiltration. Histiocytes and lymphocytes are scanty. Which of
the following
is the approximate period between the onset of pain (ie, beginning of ischemic injury) and death?

A. 1 hour

B. 12 hours

C. 2 days
33

D. 5 days

E. 10 days

Explanation:

The correct answer is C. Following irreversible ischemic injury, the heart (and any other organ)
displays an
orderly sequence of events that progresses from necrosis of parenchymal cells to inflammatory
reaction,
granulation tissue, and scar healing.

Although ischemic injury manifests with pain almost immediately following vascular occlusion,
histologic
evidence of necrosis lags behind the clinical symptoms. At 1 hour (choice A) after ischemia, there is
no
morphologic change indicative of necrosis. The first signs of necrosis appear 12 hours (choice B)
after
irreversible ischemia: myocytes appear intensely eosinophilic and wavy, but there is no inflammatory
reaction
yet. Acute inflammatory cells (neutrophils) infiltrate the infarcted area beginning 1 day and peaking
at
approximately 2-3 days after injury. This acute inflammatory response partially overlaps with the
subsequent
influx of lymphocytes and histiocytes. Reabsorption of necrotic myofibers by histiocytes, as well as
proliferation
of small blood vessels, marks early formation of granulation tissue at around 5 days (choice D).
Granulation
tissue is advanced at 10 days (choice E) and consists of fibroblasts, small blood vessels, and residual
chronic
inflammatory cells within a matrix of young collagen matrix.

A 54-year-old man presents with a chief complaint of "burning" abdominal pain in the epigastric region.
Endoscopy demonstrates a well-defined, regular gastroesophageal junction located 3 cm above the
esophageal
hiatus in the diaphragm. Biopsy of the distal side of the junction demonstrates normal gastric mucosa.
This lesion
is best classified as which of the following?

A. Achalasia

B. Esophageal ring

C. Esophageal web

D. Paraesophageal hernia

E. Sliding hernia
34

Explanation:

The correct answer is E. This patient has a sliding hiatal hernia, which is the most common (90%)
form of hiatal
hernia. This condition is frequently associated with gastric reflux.

Achalasia (choice A) is actually a physiologic, rather than an anatomic variation. In this disorder, the
lower
esophageal sphincter fails to relax adequately, and esophageal peristalsis is often abnormal.

Esophageal rings (choice B) are mucosal folds in the esophagus. They are called esophageal webs
(choice C)
in the upper esophagus. Schatzki rings are mucosal rings in the lower esophagus, at the
gastroesophageal
junction.

In a paraesophageal hernia (choice D), an area of gastric cardia rolls along with the esophagus
through an
incompetent hiatus into the thorax. In a paraesophageal hernia, the gastroesophageal junction would
not be
displaced.

Which of the following pulmonary conditions is associated with widespread formation of hyaline
membranes in the
alveolar cavities?

A. Asthma

B. Bacterial pneumonia

C. Desquamative interstitial pneumonitis

D. Diffuse alveolar damage

E. Hemodynamic pulmonary edema

Explanation:

The correct answer is D. Diffuse alveolar damage, clinically referred to as adult respiratory distress
syndrome
(ARDS), is characterized by diffuse damage to the alveolar/capillary barrier, which may result from
diverse
acute conditions. The four most frequent causes are trauma, sepsis, shock, and gastric aspiration. The
pathogenesis is not entirely clear, but influx of neutrophils and release of cytokines, eicosanoids, and
free
radicals seem to be crucial in promoting alveolar damage. The most characteristic histopathologic
hallmark of
diffuse alveolar damage is formation of hyaline membranes within the alveolar cavities. These consist
of
proteinaceous material of plasma origin and necrotic debris from desquamated epithelium. The
condition has a
35

60% mortality and manifests with acute respiratory failure.

The pathologic features of asthma (choice A) are relatively nonspecific and are similar, for example,
to those of
chronic bronchitis, including chronic inflammatory infiltration, hyperplasia of mucous glands, and
hypertrophy of
smooth muscle. The presence of numerous eosinophils, however, is more characteristic of asthma.

Bacterial pneumonia (choice B) is characterized by intra-alveolar exudation of neutrophils, fibrin, and


erythrocytes. Bacteria are also present within the alveolar cavities.

Desquamative interstitial pneumonitis (DIP) (choice C) is a form of interstitial disease referred to as


idiopathic
pulmonary fibrosis. In contrast to usual interstitial pneumonitis (another form of idiopathic pulmonary
fibrosis),
DIP is more responsive to steroid treatment. Histopathologically, DIP leads to hyperplasia of
pneumocytes and
accumulation of histiocytes that fill the alveolar cavities.

Hemodynamic pulmonary edema (choice E) is caused by increased hydrostatic pressure, as occurs in


acute left
ventricular failure. It is due to escape of fluid from the intravascular compartment into the alveoli.

A baby is born with a flat facial profile, prominent epicanthal folds, and simian crease. She vomits when
fed, and
upper GI studies demonstrate a "double bubble" in the upper abdomen. Which of the following
cardiovascular
abnormalities might this child also have?

A. Atrial septal defect

B. Berry aneurysm

C. Coarctation of the aorta

D. Endocardial cushion defect

E. Tetralogy of Fallot

Explanation:

The correct answer is D. The disease is Down syndrome (trisomy 21). In addition to mental
retardation and the
characteristic physical findings described in the question stem, duodenal atresia is fairly common, as
evidenced
by the "double bubble" sign on x-ray. These children are also likely to have various cardiac
anomalies;
endocardial cushion defect is the most common.

Atrial septal defect (choice A) is one of the most common genetic defects in the general population,
but is less
36

common than endocardial cushion defect in patients with Down syndrome.

Berry aneurysms (choice B), also known as saccular aneurysms, are typically located in the circle of
Willis on
the ventral surface of the brain. They occur more frequently in patients with adult polycystic disease.
Rupture
can produce subarachnoid hemorrhage.

Coarctation of the aorta (choice C) occurs more commonly in females with a 45, XO genotype
(Turner
syndrome).

Tetralogy of Fallot (choice E) is the most common cause of early cyanosis, consisting of a ventricular
septal
defect, right ventricular outflow tract obstruction, an overriding aorta, and right ventricular
hypertrophy.

A 47-year-old man presents to the emergency room with sudden onset of severe upper abdominal pain with
vomiting. The pain is focused in the epigastrium with radiation to the back. Serum amylase levels are
2000 U/L.
Which of the following are the most commonly encountered predisposing factors for this patient's
condition?

A. Alcohol use and gallstones

B. Helicobacter pylori infection and excess gastric acid secretion

C. Hepatitis B infection and iron overload

D. Obesity and high serum cholesterol

E. Stress and cigarette use

Explanation:

The correct answer is A. The clinical scenario is typical of acute pancreatitis. The overwhelmingly
most
important contributing factors for development of acute pancreatitis are gallstones (particularly small
ones) and
alcohol abuse.

Helicobacter pylori infection and excess gastric acid secretion (choice B) are predisposing factors for
peptic
ulcer disease of the stomach and duodenum, respectively.

Hepatitis B infection and iron overload (choice C) predispose for cirrhosis.

Predisposing factors for myocardial infarction include obesity, high serum cholesterol (choice D),
stress, and
cigarette smoking (choice E).
37

A 37-year-old woman complains to her gynecologist of discomfort during intercourse and placement of a
tampon.
Physical examination demonstrates a flocculent swelling below the skin of the posterolateral part of
one labium
majora.Which of the following is the most likely diagnosis?

A. Bartholin's gland cyst

B. Condyloma acuminatum

C. Lichen sclerosis

D. Vestibular adenitis

E. Vulvar squamous hyperplasia

Explanation:

The correct answer is A. This is a Bartholin's gland cyst, which is a relatively common lesion
occurring when
Bartholin's duct becomes obstructed, typically a sequela to a previous infection. The cysts can enlarge
to 3 to 5
cm in diameter. They are lined by either transitional epithelium or metaplastic squamous epithelium.
Treatment
is by excision or marsupialization (permanent opening).

Condyloma acuminatum (choice B) usually produces a papillary lesion (venereal wart).

Lichen sclerosis (choice C) usually produces a gray, parchment-like thinned epidermis.

Vestibular adenitis (choice D) usually produces an exquisitely tender posterior introitus with focal
ulcerations.

Vulvar squamous hyperplasia (choice E) usually produces a white plaque.

Which of the following types of hepatitis is associated with an immune-mediated vasculitis characterized
by
p-ANCA antibodies?

A. Hepatitis A

B. Hepatitis B

C. Hepatitis C

D. Hepatitis D

E. Hepatitis E

Explanation:
38

The correct answer is B. Hepatitis B is associated with polyarteritis nodosa (PAN), a necrotizing
vasculitis of
small- and medium-sized muscular arteries involving all organ systems. A significant percentage of
patients with
PAN have hepatitis B antigenemia. They also have circulating immune complexes containing
hepatitis B antigen.
Hepatitis B antigen, IgM, and complement can be demonstrated in blood vessel walls. P-ANCA
(perinuclear-antinuclear cytoplasmic antibody) is a marker for polyarteritis nodosa.

Hepatitis A (choice A) is not associated with a vasculitis.

Hepatitis C (choice C) accounts for 50-70% of chronic hepatitis. Chronic hepatitis C can be
associated with
immune-complex mediated extrahepatic complications, but is less common than hepatitis B. Hepatitis
C does
have a significant association with essential mixed cryoglobulinemia, which presents with
glomerulonephritis,
arthralgias, hepatosplenomegaly, and lymphadenopathy, in addition to a vasculitis. However, there is
no
association with p-ANCA.

Hepatitis D (choice D) requires that the patient be co-infected with hepatitis B. As such, it does not
independently cause disease, but it can produce a worsening of the liver disease.

Hepatitis E (choice E) resembles hepatitis A due to its primarily enteric mode of spread. It is not
associated with
chronic hepatitis and does not have a predisposition for vasculitis.

Testicular biopsy of an infertile man demonstrates a complete absence of sperm or sperm precursors in
spermatic tubules that have a regular, round cross-section and are closely packed together. The most
probable
etiology is

A. diabetes mellitus

B. maturation arrest

C. seminoma

D. Sertoli-only syndrome

E. tuberculosis

Explanation:

The correct answer is D. All of the conditions listed can cause infertility due to a low or absent sperm
count.
Only in Sertoli-only syndrome is there a complete absence of sperm precursors in an undamaged
tubule.
39

There is no known method to correct Sertoli-only syndrome (or maturation arrest) that is not due to a
treatable
chronic disease.

Chronic diseases such as diabetes mellitus (choice A) or tuberculosis (choice E) can arrest the
maturation of
sperm, but do not usually show a complete absence of sperm precursors.

In maturation arrest (choice B), mature sperm are absent, but precursors are found.

Tumors such as seminomas (choice C) cause infertility by occluding the flow of semen or by
replacing the
seminiferous tubules. Sampling of a seminiferous tubule not replaced by tumor would probably still
demonstrate
sperm.

A 50-year-old woman who works as a paralegal in a law firm comes to her local doctor because of
problems with
sleep. The patient says that over the past several weeks, she hasn't slept well, feels tired, and has had
headaches. She does not smoke or drink alcohol, except on special occasions, and does not take any
medications. The patient's pupils are 5 mm in size, equal and reactive, with both the direct and
consensual light
reflexes intact. Accommodation is unimpaired. Examination of the visual fields and funduscopy are
unremarkable.
Extraocular movements reveal normal conjugate, oblique, and downward movement, but she is unable
to look
upwards. No other abnormalities are found on the neurological examination. Which of the following is
the most
likely diagnosis?

A. Acoustic neuroma

B. Astrocytoma in the cerebellum

C. Craniopharyngioma

D. Parasagittal meningioma

E. Pinealoma

Explanation:

The correct answer is E. This patient has a pinealoma. Tumors of the pineal gland compress the
vertical gaze
center in the tectum of the midbrain. The pineal gland manufactures melatonin from its precursor
serotonin; an
inadequate supply of melatonin results in insomnia. Tumors of the pineal gland will not compress the
cerebral
cortex or the rest of the brainstem. Frequently, the only physical sign noted is failure of upward gaze.
40

An acoustic neuroma (choice A) is a schwannoma of the eighth cranial nerve. It results in deafness,
ataxia, and
dysarthria. Nystagmus may be present. The gaze centers are not affected.

Astrocytomas of the cerebellum (choice B) are usually seen in children. These tumors present with
headache,
nausea, vomiting, papilledema, and cerebellar signs such as ataxia, dysarthria, nystagmus, and
intention
tremor. The gaze centers are not affected.

Craniopharyngiomas (choice C) are usually seen in children. There is failure of growth, headaches,
and
bitemporal hemianopia.

Parasagittal meningiomas (choice D) usually result in headache, spastic paresis, and urinary
incontinence.

A 25-year-old man experiences the gradual onset of intermittent diarrhea, which over years, progresses to
severe diarrhea, alternating with constipation, rectal bleeding, and passage of mucus. On physical
examination,
the abdomen is tender over the colon. Stool examination fails to reveal parasites. Colonoscopy
demonstrates
inflammation limited to the rectum, with no higher lesions. Which of the following diseases would
most likely be
seen in a close relative of the patient?

A. Celiac disease

B. Crohn's disease

C. Hirschsprung's disease

D. Tropical sprue

E. Whipple's disease

Explanation:

The correct answer is B. The presentation is classic for ulcerative colitis. Family members have an
increased
incidence of both ulcerative colitis and Crohn's disease, supporting the idea that these two diseases are
actually different ends of the same spectrum. In contrast to Crohn's disease, in which the lesions may
be patchy
and involve the distal ileum and even the esophagus, in ulcerative colitis, the lesions involve the
rectum and
may extend continuously proximally for varying distances up to the cecum and very distal end of the
ileum.

Celiac disease (choice A) is a small intestinal disease related to gluten intolerance. Flattening of villi,
elongated
crypts and marked inflammation in the lamina propria are noted histologically.
41

Hirschsprung's disease (choice C) is a congenital cause of severe constipation and megacolon due to a
lack of
ganglion cells in the distal colon.

Tropical sprue (choice D) clinically resembles celiac disease, but may be related to infection.

Whipple's disease (choice E) is an intestinal diarrheal disease that has been shown to be due to a
bacterial
infection.

A surgeon operating on a 47-year-old woman finds cystic masses in both ovaries. Each mass consists of a
unilocular cyst containing clear fluid. The entire wall is covered with papillary excrescences. Which of
the following
is the most likely diagnosis?

A. Endometrioid adenocarcinoma

B. Granulosa cell tumor

C. Mature cystic teratoma

D. Mucinous cystadenocarcinoma

E. Serous cystadenocarcinoma

Explanation:

The correct answer is E. Similar to testicular tumors, ovarian tumors can be classified according to cell
of origin.
There are three main categories: tumors of surface epithelium, tumors of germ cell origin, and tumors
of sex
cord-stromal origin. Ovarian surface epithelium (coelomic mesothelium) may differentiate along tubal
(serous),
cervical (mucinous), or endometrial lines, giving origin to serous cystadenoma/cystadenocarcinoma,
mucinous
cystadenoma/cystadenocarcinoma and endometrioid tumors, respectively. Serous tumors represent
40% of all
ovarian tumors, and serous cystadenocarcinoma is the most frequent serous tumor. Serous
cystadenocarcinomas occur primarily in women aged 40 to 65 years. About two thirds of these tumors
are
bilateral. As the name suggests, it is a cystic tumor containing clear fluid. The cystic wall is lined by
malignant
epithelial cells forming papillary fronds.

Less frequent than serous cystadenocarcinoma, endometrioid adenocarcinoma (choice A) and


mucinous
cystadenocarcinoma (choice D) also derive from surface epithelium. Endometrioid carcinoma is
histologically
similar to endometrial adenocarcinoma, whereas mucinous cystadenocarcinoma is composed of
mucin-producing
cells similar to cervical epithelium. Both these neoplasms have solid and cystic areas (mucinous
42

cystadenocarcinoma more so than endometrioid carcinoma) and may be bilateral (endometrioid


carcinoma more
frequently than mucinous cystadenocarcinoma).

Granulosa cell tumors (choice B) originate from ovarian stroma and consist of variable mixtures of
granulosa
cells and theca cells. Since they frequently produce large amounts of estrogens, these tumors manifest
with
precocious puberty in preadolescent girls. On the contrary, mature women with granulosa cell tumors
develop
endometrial hyperplasia and fibrocystic change of breast. Histologically, these neoplasms are
composed of
uniform cuboidal cells, forming structures reminiscent of ovarian follicles (Call-Exner bodies).

Mature cystic teratoma (choice C) is the most frequent neoplasm derived from germ cells. Teratomas
can be
further classified into mature cystic, immature, and monodermal teratomas. The great majority of
teratomas are
mature cystic. Since they originate from more than one germ layer, these neoplasms contain an
amazing mixture
of mature tissue components, often including skin, teeth, neural epithelium, thyroid, cartilage, and
intestinal
tissue, for example.

A 24-year-old woman is seen by her family practitioner. Her urine sample has a stable, frothy white foam
on top.
Which of the following substances is likely to be present in her urine in significant amounts?

A. Bilirubin

B. Blood

C. Glucose

D. Ketones

E. Protein

Explanation:

The correct answer is E. Reagent strips of various types are commonly used both in physicians'
offices and in
hospital laboratories for rapid semiquantitative urinalysis. The Multistix strip, which is one of the
more commonly
used strips, contains reagent squares for glucose, bilirubin, ketones, specific gravity, blood, pH,
protein,
urobilinogen, nitrite, and leukocytes. Each of these squares undergoes a chemical change when
dipped in
urine, causing the color of the square to change. The result is "read" by comparing the new color to
reference
colors on the bottle. In this case, you need to know that a stable froth on urine is usually due to
proteinuria
43

(more than several grams per 24 hr); therefore, the protein indicator would be positive on the dipstick.

High levels of bilirubin (choice A) in urine can cause an unusual yellow foam.

Blood in the urine (choice B) might be present in some forms of renal disease, but would not explain
the stable
foam.

High levels of glucose (choice C) in urine can cause it to develop a sweet smell and taste; smelling
and tasting
urine was an ancient method of diagnosing diabetes mellitus, but is no longer recommended for
obvious
reasons.

Ketones (choice D) may give urine an acetone-like odor, but testing for ketones in this manner is no
longer
recommended for obvious reasons.

A patient with a long-standing intrauterine contraceptive device develops chronic pelvic pain. The device is
removed, and a biopsy of the endometrium is performed. The biopsy specimen shows a prominent
infiltrate
composed of lymphocytes, plasma cells, and histocytes. Which of the following is the most likely
diagnosis?

A. Acute endometritis

B. Adenomyosis

C. Chronic endometritis

D. Endometriosis

E. Simple hyperplasia of endometrium

Explanation:

The correct answer is C. This is chronic endometritis, evidenced by the chronic inflammatory
infiltrate of
lymphocytes, plasma cells, and histiocytes. This disorder may be idiopathic but is more often
associated with an
obvious predisposing factor, such as chronic pelvic inflammatory disease, tuberculosis, retained
gestational
tissue, or, as in this case, an intrauterine contraceptive device. Chronic endometritis can cause
abnormal
bleeding, pain, and infertility.

Acute endometritis (choice A) is characterized by a prominent neutrophilic infiltrate and usually


occurs after
delivery or miscarriage.

Adenomyosis (choice B) refers to endometrium abnormally located in myometrium.


44

Endometriosis (choice D) refers to abnormally located patches of endometrium (except in the


myometrium,
where it would be called adenomyosis).

Simple hyperplasia of endometrium (choice E) causes cystically dilated glands in endometrium.

Autopsy of a 72-year-old man demonstrates the presence of deep venous thrombosis of the legs and
multiple
acute brain infarcts due to thromboembolic occlusion of penetrating arteries. Which of the following
pathologic
conditions would most likely account for these findings?

A. Atherosclerosis of penetrating cerebral arteries

B. Endocarditis of the tricuspid valve

C. Patent foramen ovale

D. Pulmonary thromboembolism

E. Trousseau syndrome

Explanation:

The correct answer is C. Persistence of patent foramen ovale is found in a significant proportion of
healthy
subjects. A widely patent foramen ovale may allow emboli originating from the veins in the legs to
bypass the
pulmonary circulation and reach the systemic arteries, thereby producing infarcts (paradoxical
embolism) in the
brain as well as in other organs. Interatrial or interventricular defects can have the same effect. None
of the
other answer choices would explain the development of embolic infarcts in the cerebral parenchyma.

Atherosclerotic changes are frequently found in the circle of Willis and its major branches, but not in
the
small-caliber penetrating arteries of the brain (choice A).

Endocarditis of the tricuspid valve (choice B) may give rise to emboli resulting from fragmentation of
valvular
vegetations. Emboli from the tricuspid valve, however, would enter the pulmonary circulation,
possibly leading to
infarcts of the lungs.

Pulmonary thromboembolism (choice D) frequently occurs as a result of deep venous thrombosis,


especially
after immobilization, bed rest, obstetric delivery, and surgery. However, thromboemboli that become
lodged in
the pulmonary arteries cannot pass through the pulmonary capillary filter and cause systemic
embolization.
45

Trousseau syndrome (choice E), also known as migratory thrombophlebitis, occurs in association
with
disseminated cancers, especially mucinous adenocarcinomas.This condition is probably due to release
of
procoagulant factors by the tumor and manifests with recurrent episodes of thrombosis affecting veins
(but not
arteries) in both limbs and visceral organs.
1

A patient complains of difficulty breathing through his nose and bony pain in his cheeks, near his nose.
Physical examination and CT of the head reveal mass lesions involving the nose, pharynx, and sinuses. CT-
guided biopsy demonstrates a non-keratinizing, squamous cell carcinoma. Which of the following disorders
is associated with the same oncogenic virus that is the likely cause of this patient's cancer?

A. Adult T-cell leukemia

B. Burkitt's lymphoma

C. Cervical carcinoma

D. Hepatocellular carcinoma

E. Kaposi's sarcoma
Explanation:
The correct answer is B. The disease is nasopharyngeal carcinoma, which is associated with the Epstein-
Barr Virus (EBV). This virus is also associated with the African form of Burkitt's lymphoma that
characteristically involves the jaw.
HTLV-1, or human T-lymphocyte virus, is associated with adult T-cell leukemia (choice A).
HPV, or human papillomavirus, is associated with cervical carcinoma (choice C), penile carcinoma, and
anal carcinoma.
Hepatitis B virus (HBV) is associated with hepatocellular carcinoma (choice D). HHV 8, a member of the
herpes family, is associated with Kaposi's sarcoma (choice E).

A 34-year-old man develops pulmonary hemorrhage and glomerulonephritis. Lung biopsy with
immunofluorescence demonstrates IgG deposition along the basement membrane. These antibodies
are most likely directed against which of the following types of collagen?

A. Type I

B. Type II

C. Type III

D. Type IV

E. Type X
Explanation:
The correct answer is D. The disease described is Goodpasture's syndrome, in which autoantibodies to
basement membrane proteins cause damage to the lungs and kidneys. Pulmonary hemorrhage (especially
in smokers) and rapidly progressive glomerulonephritis are common. The characteristic autoantibody
present is directed against Type IV collagen, a component of the basement membrane.
Type I collagen (choice A) is found in bone, skin, tendon, dentin, fascia, and late wound repair.
Type II collagen (choice B) is found in cartilage (including hyaline cartilage), the vitreous body of the eye,
and the nucleus pulposus of the intervertebral disk. Type III collagen (choice C) is found in skin, blood
vessels, uterus, fetal tissue, and granulation tissue.
Type X collagen (choice E) is found in epiphyseal plates.

A patient has long-standing severe hemolytic anemia characterized by hypochromic cells. Electrophoresis
studies demonstrate a near complete absence of beta chains. Several years later, the patient develops
cardiac failure. Intracardiac deposition of which of the following would be most likely to contribute to the
cardiac failure?
2

A. Calcium

B. Iron

C. Magnesium

D. Potassium

E. Sodium
Explanation:
The correct answer is B. The disease is beta thalassemia major, which is a severe hemolytic anemia
characterized by a failure to produce the beta chains of hemoglobin (some HbF, the fetal form of
hemoglobin, is produced). The excess alpha chains are insoluble, leading to intra- and extravascular
hemolysis. These patients require large numbers of transfusions, and iron overload with resulting secondary
hemochromatosis can contribute to eventual cardiac failure. The heart is also damaged by the chronic high
output state needed to compensate for the anemia.
Calcium (choice A) deposition is seen in damaged tissues and states with high serum calcium, such as
hyperparathyroidism.
Magnesium (choice C), potassium (choice D), and sodium (choice E) are highly soluble and do not usually
precipitate in tissues.

A 30-year-old woman presents with complaints of weakness and headaches. Her friends say she has been
irritable and depressed lately. On physical examination, the patient is jaundiced, and her liver is small and
firm. Neurologic examination is remarkable for choreoathetotic movements and a fine tremor that, when
her upper limbs are extended, resembles a bird flapping its wings. Which of the following tests would most
likely lead to to correct diagnosis?

A. Nerve conduction studies

B. Prussian blue stain of liver biopsy specimen

C. Serum alkaline phosphatase

D. Serum transaminases

E. Slit-lamp examination of the eyes

Explanation:

The correct answer is E. This patient is exhibiting symptoms of Wilson's disease, which is due to
inadequate
copper excretion by the biliary system. Mutations in a copper-transporting ATPase, coded for by the
ATP7B
gene on chromosome 13, appear to underlie this autosomal recessive disorder. Accumulation of
copper in the
liver initially produces fatty change, followed by hepatocellular necrosis, inflammation, bile duct
proliferation,
and cirrhosis. Eventually, the copper spills out of the liver to deposit in other tissues, notably the
brain. The
caudate nucleus and putamen are generally most affected; injury to these structures produces an
extrapyramidal movement disorder that most commonly presents with choreoathetosis and tremor but
which
3

may produce cerebellar signs or parkinsonism. Various psychiatric symptoms can accompany
neurological
involvement. Copper deposition in Descemet's membrane in the cornea produces the nearly
pathognomonic
Kayser-Fleischer ring, which can be seen with slit-lamp examination of the eyes. Failure to
demonstrate a
Kayser-Fleischer ring in a patient with hepatic disease and neurological impairment virtually excludes
the
diagnosis of Wilson's disease. Low serum ceruloplasmin and increased urinary copper, or increased
copper
levels on liver biopsy, are diagnostic.

Nerve conduction studies (choice A) would be of little value in the diagnosis of Wilson's disease,
although they
are valuable in detecting dysfunction of peripheral nerves that result, for example, from
demyelination, loss of
nerve axons, failure of conduction, or neuromuscular junction failure.

A Prussian blue stain performed on a liver biopsy (choice B) would demonstrate increased iron stores
in a
patient's liver secondary to hemochromatosis.

Serum alkaline phosphatase (choice C) would be elevated in patients with biliary tract disease,
although other
types of liver disease, bone disease, or pregnancy can produce elevations as well.

Serum transaminases (choice D) would be increased in patients with a variety of diseases causing
hepatocellular injury, including hepatitis and cirrhosis, but this finding is not particularly specific.

A physician in the emergency department is evaluating a patient with severe chronic obstructive pulmonary
disease
prior to oxygen supplementation. The physician decides to draw arterial blood for blood gas studies.
His technique
is faulty, however, and he introduces room air into the syringe while pulling on the plunger as he is
drawing the
syringe out of the patient. Which of the following patterns of changes would be most likely to be
produced by this
exposure of arterial blood to room air?

PO2
PCO2
pH

A. Decreased
Decreased
Decreased

B. Decreased
Elevated
Decreased

C. Elevated
4

Decreased
Decreased

D. Elevated
Decreased
Elevated

E. Elevated
Elevated
Elevated

Explanation:

The correct answer is D. The technical part of the collection of arterial blood samples is difficult.
Some hospitals
allow only physicians to collect the samples, while other hospitals allow nurses or technicians with
additional
special training to collect the samples. No matter who performs the arterial draw, care must be taken to
avoid
exposing the blood to room air, as such exposure tends to cause the blood to partially equilibrate with
the room
air. Room air would have a higher PO2 and a lower PCO2 than this patient's blood, so the sample
would have a
higher PO2 and a lower PCO2. In the atmosphere, PO2 = 150 mm Hg and PCO2 is near 0 mm Hg; in
the arterial
blood of a healthy patient, PO2 = 100 mm Hg, PCO2 = 40 mm Hg (PO2 could be lower and PCO2
higher in a
diseased individual). Because CO2 is decreased, there will be less carbonic acid present in the blood,
thus
raising the pH.

In which of the following sites do myxopapillary ependymomas most frequently occur?

A. Cerebellum

B. Conus medullaris

C. 4th ventricle

D. Lateral ventricles

E. Midbrain

Explanation:

The correct answer is B. Myxopapillary ependymoma is a variant of ependymoma, a tumor arising


from
ependymal cells. Histologically, myxopapillary ependymoma contains a myxoid (mucus-rich)
intercellular matrix,
5

in which spindly neoplastic ependymal cells are arranged in a fascicular and papillary pattern (hence
its
designation). It is a benign tumor that almost always occurs in the distal segment of the spinal cord,
ie, the
conus medullaris. Once excised, the patient is cured.

The cerebellum (choice A) is the favorite site for pilocytic astrocytomas, medulloblastomas, and
hemangioblastomas, but not ependymomas.

In general, classic ependymomas occur in close proximity to the ventricular cavities, specifically, the
4th
ventricle (choice C) in children and the lateral ventricles (choice D) in adults. The myxopapillary
variant does
not occur in either location.

A midbrain location (choice E) would be truly exceptional for any type of ependymoma.

A 54-year-old woman with chronic microcytic hypochromic anemia also has a sore, smooth, red tongue
and a
sense of dysphagia midway during swallowing. This patient is at increased risk for developing which
of the
following conditions?

A. Adenocarcinoma of the esophagus

B. Barrett's esophagus

C. Candida esophagitis

D. CMV esophagitis

E. Squamous cell carcinoma of esophagus

Explanation:

The correct answer is E. The patient has Plummer-Vinson syndrome, characterized by atrophic
glossitis,
esophageal webs, and iron-deficiency anemia. Patients with this syndrome are at increased risk of
developing
squamous cell carcinoma of the esophagus.

Barrett's esophagus (choice B) and adenocarcinoma of the esophagus (choice A) are associated with
reflux
esophagitis.

Candida(choice C) and CMV (choice D) esophagitis can be seen in immunosuppressed patients,


including AIDS
patients.
6

Which of the following locations is most likely for the development of carcinoma in a 32-year-old baseball
player
who has chewed tobacco for 15 years?

A. Floor of the mouth

B. Lower lip

C. Tongue

D. Tonsils

E. Upper lip

Explanation:

The correct answer is B. Oral cancer is most strongly related to tobacco chewing, with weaker
associations with
cigarette smoking, pipe smoking, and alcohol use. Unfortunately, many teenagers believe that
chewing tobacco
is "harmless" because it does not cause lung cancer, and the case illustrated in the question is
unfortunately
not uncommon. Oral cancers tend to occur on the lower lip (40%; choice B), tongue (20%; choice C),
floor of
the mouth (15%; choice A), with other oral sites (choices D and E) being less common. They are
usually
squamous cell carcinomas and unlike their skin counterparts, frequently cause both extensive
morbidity and
mortality.

A 67-year-old male smoker presents to his physician for a routine physical examination. Chest x-ray
demonstrates
a 2-cm density on the left side. Laboratory studies are remarkable for a serum sodium of 134 mEq/L.
The findings
may be attributable to tumor cell secretion of

A. adrenocorticotrophic hormone (ACTH)

B. antidiuretic hormone (ADH)

C. melanocyte-stimulating hormone (MSH)

D. parathyroid hormone (PTH)

E. vasoactive intestinal polypeptide (VIP)

Explanation:

The correct answer is B. All of the hormones listed can be secreted by bronchogenic carcinoma, and
may cause
7

a paraneoplastic syndrome. Of the answer choices provided, only ADH (antidiuretic hormone) causes
hyponatremia.

ACTH (choice A) causes Cushing's syndrome.

MSH (choice C) causes increased skin pigmentation.

PTH (choice D) causes hypercalcemia.

VIP (choice E) causes diarrhea and hypokalemia.

Other hormones that can be produced include human chorionic gonadotropin (hCG; gynecomastia),
prolactin
(lactation), and calcitonin (hypocalcemia).

A 48-year-old female presents to the doctor with lower back pain. She states that she has had the pain for
about
two weeks and that it has become steadily more severe. An x-ray shows a lytic bone lesion in her
lumbar spine.
Review of systems reveals the recent onset of mild headaches, nausea, and weakness. Her CBC shows
a
normocytic anemia, and her erythrocyte sedimentation rate is elevated. Urinalysis shows heavy
proteinuria, and a
serum protein electrophoresis shows a monoclonal peak of IgG. Which of the following is responsible
for this
patient's spinal lesion?

A. Bence-Jones proteins

B. Lymphoplasmacytoid proliferation

C. Osteoblast activating factor

D. Osteoclast activating factor

E. Primary amyloidosis (AL)

Explanation:

The correct answer is D. First of all, the disease described above is multiple myeloma. Multiple
myeloma is a
plasma cell neoplasm in which the plasma cells proliferate a single, or monoclonal, type of
immunoglobulin. In
this case, and most commonly, IgG is produced. Patients with this disease are usually over 40 and
may have
normocytic anemias. They often complain of skeletal pain from lytic bone lesions and may report
headaches
and nausea caused by hyperviscosity of the blood due to the excessive amounts of immunoglobulins.
The lytic
bone lesions are caused by the production of osteoclast activating factor by the neoplastic plasma
cells. This
can also lead to hypercalcemia.
8

Bence-Jones proteins (choice A), are immunoglobulin light chains. They are often overproduced in
multiple
myeloma and are filtered in the urine. They are not usually detected in serum unless there is renal
impairment,
but they can be detected in the urine by electrophoresis and immunofixation. They do not cause bony
lytic
lesions.

Lymphoplasmacytoid proliferation (choice B), describes a normal type of B lymphocyte which is


morphologically
between a lymphocyte and a plasma cell. Lymphoplasmacytoid lymphocytes produce IgM, and in
Waldenstrom's
macroglobulinemia, they undergo neoplastic proliferation and produce IgM peaks. Bone lesions are
not seen in
this disease.

Osteoblast activating factor (choice C), would not produce osteolytic lesions and is not seen in
multiple
myeloma. There is a rare osteoblastic variant of multiple myeloma with dense bony osteosclerosis
rather than
lytic lesions, but osteoblast activating factor has not been shown to be involved.

Primary amyloidosis (AL) (choice E), is a primary light-chain type of amyloidosis associated with
multiple
myeloma. The insoluble proteinaceous deposits occur in the tongue, heart, kidney, and skin. This does
not
cause bony lytic lesions.

A 35-year-old man who recently traveled to a third world country develops chronic, severe dysentery.
Colonoscopy demonstrates ulceration of the cecum, and a cecal biopsy reveals 15-to-40 micron
amoebae with
ingested erythrocytes and small nuclei with distinctive tiny central karyosomes. Which of the
following organisms
is the most likely culprit?

A. Acanthamoeba sp.

B. Balantidium coli

C. Entamoeba histolytica

D. Giardia lamblia

E. Naegleria fowleri

Explanation:

The correct answer is C.Entamoeba histolytica is the usual cause of intestinal amebiasis, and has the
microscopic features described in the question stem. A particularly helpful (but not always present)
feature of
this organism is the presence of ingested red blood cells within the amoebae. These amoebae cause
9

flask-shaped ulceration of the intestinal mucosa and submucosa, with a particular propensity for
involving the
cecum and ascending colon. The disease manifestations range from none (asymptomatic carriers) to
mild
chronic diarrhea, to severe, purging dysentery. In symptomatic cases, the liver may develop
destructive
amoebic liver abscesses that tend to become secondarily (and potentially life-threateningly) infected
by
bacteria.

Acanthamoeba(choice A) is a free-living amoebae that can cause amoebic meningocephalitis.

Balantidium coli(choice B) is a large ciliated intestinal parasite that can occasionally cause colonic
disease
resembling that caused by Entamoeba histolytica.

Giardia lamblia(choice D) is a small intestinal protozoa with a distinctive pear-shaped morphology


that appears
to have a "face."

Naegleria fowleri(choice E) is a free-living amoebae that can cause amoebic meningoencephalitis.

A 27-year-old man develops bilateral parotid gland swelling and orchitis, and is generally ill with fever of
102° F.
Which of the following substances is most likely to be significantly elevated in the patient's serum?

A. Alanine aminotransferase (ALT)

B. Amylase

C. Aspartate aminotransferase (AST)

D. Ceruloplasmin

E. Creatine phosphokinase, MB isoenzyme (CPK-MB)

Explanation:

The correct answer is B. The disease is mumps, caused by a paramyxovirus. In children, mumps
causes a
transient inflammation of the parotid glands, and less commonly, the testes, pancreas, or central
nervous
system. Mumps tends to be a more severe disease in adults than in children. Mumps in adults involves
the
testes (causing orchitis) and pancreas with some frequency. Pancreatic involvement can cause
elevation of
serum amylase.

ALT (choice A) and AST (choice C) are markers for hepatocellular damage.

Ceruloplasmin (choice D) is a copper-carrying protein that is decreased in Wilson's disease.


10

CPK-MB (choice E) is the isoenzyme of CPK that is relatively specific for the myocardium. This
enzyme is
increased in the early stages of a myocardial infarction.

A 50-year-old hypertensive man develops very severe, "tearing" chest pain, which migrates from his upper
back
to mid-back over the period of an hour. Pathologic examination of a specimen removed from the
patient during
emergency surgery would most likely demonstrate which of the following?

A. Cystic medial degeneration

B. Infarction

C. Plasma cells around the vasa vasorum

D. Severe atherosclerosis

E. Tree-barking

Explanation:

The correct answer is A. This is a classic description of a dissecting aortic aneurysm, a very important
condition
that may cause death if missed or misdiagnosed. Dissecting aneurysms are actually dissecting
hematomas, with
the blood located between the middle and outer thirds of the media of the aorta. Dissecting aneurysms
are
associated with hypertension in many cases; they are also associated with cystic medial degeneration
of the
wall of the aorta (seen in Marfan's syndrome). Often, an intimal tear is present; these are thought to
represent
the starting point for the dissection. Unlike abdominal aortic aneurysm and syphilitic aneurysm, aortic
dissection
is not usually associated with aortic dilatation.

The pain of myocardial infarction (choice B) does not usually move.

Plasma cells around the vasa vasorum (choice C) and "tree-barking" (wrinkling of the aortic intima;
choice E )
are features of syphilitic aneurysms.

Atherosclerotic (choice D) aneurysms typically affect the abdominal aorta.

The presence of the Philadelphia chromosome is associated with a more favorable prognosis in patients
with
which of the following diseases?

A. Acute lymphoblastic leukemia


11

B. Acute myelogenous leukemia

C. Chronic lymphocytic leukemia

D. Chronic myelogenous leukemia

E. Hairy cell leukemia

Explanation:

The correct answer is D. The presence of the Philadelphia chromosome, a translocation from the long
arm of
chromosome 22 to chromosome 9 [t(9;22)], is associated with a more favorable prognosis in patients
with
chronic myelogenous leukemia.

Acute lymphoblastic leukemia (ALL; choice A) is the most common cause of leukemia in children.
The presence
of the Philadelphia chromosome is associated with a worse prognosis for the patient. This form of
leukemia is
also associated with a B-ALL translocation of the c-myc proto-oncogene of chromosome 8 to
chromosome 14
[t(8;14)(q24;q32)].

Acute myelogenous leukemia (AML; choice B) is the most common acute leukemia in adults. The
M2 subtype is
associated with the t(8;21) translocation and the M3 subtype is associated with the t(15;17)
translocation.

Over half of patients with chronic lymphocytic leukemia (choice C) display one of several
chromosomal
abnormalities. This includes trisomy 12 (involves the h-ras proto-oncogene), translocation t(11;14)
(involves
k-ras and bcl-1 proto-oncogenes), and deletion (14q-) or inversion (14q) (involves immunoglobulin
heavy chain
gene).

Hairy cell leukemia (choice E) is associated with the expression of tartrate-resistant acid phosphatase
(TRAP)
on the surface of B cells.

A 32-year-old woman visits her gynecologist for a Pap smear. On physical examination, her gynecologist
palpates
a large adnexal mass on the right. After ultrasound confirmation of a large ovarian mass, a laparotomy
is
scheduled, and the mass is removed. Pathologic examination of the mass demonstrates a cystic cavity
filled with
hair and keratin debris, and the wall contains skin, adnexal tissue, thyroid tissue, and neural tissue. All
of the
tissues are similar to those normally found, and no malignant changes are seen. Which of the
following is the
most likely diagnosis?
12

A. Immature teratoma

B. Leiomyoma

C. Leiomyosarcoma

D. Mature teratoma

E. Rhabdomyosarcoma

Explanation:

The correct answer is D. The lesion is a mature teratoma. Teratomas located in the ovary and
containing a hair
and keratin filled cyst are sometimes called dermoid cysts. Teratomas contain cells of a variety of
types, often
including skin, skin adnexal structures (hair follicles, sweat glands, sebaceous glands), connective
tissues,
neural tissue, muscle, and thyroid tissue. If immature tissues such as primitive neuroepithelial cells or
developing skeletal muscle cells are seen, the lesion is considered potentially malignant and classified
as an
immature teratoma (choice A).

Leiomyomas (choice B) are benign tumors of smooth muscle (e.g., uterine "fibroids"), usually in the
female
genital tract.

Leiomyosarcomas (choice C) are rare malignant tumors of smooth muscle, usually in the female
genital tract.

Rhabdomyosarcomas (choice E) are malignant skeletal muscle tumors with a predilection for the head
and neck
and urogenital regions in children.

An endocrinologist examines a patient suspected of having Riedel thyroiditis. Which of the following
findings on
physical examination would best help confirm the diagnosis?

A. Eyeball protrusion

B. Massive soft thyroid gland

C. Single large thyroid nodule

D. Very tender and painful thyroid

E. "Woody" thyroid gland

Explanation:
13

The correct answer is E. Riedel thyroiditis, also called ligneous (rocklike) stroma, is a rare form of
chronic
thyroiditis characterized microscopically by a marked fibrous reaction that destroys most or all of the
thyroid
gland and may involve adjacent structures. The etiology is unknown. Clinically, this disease tends to
affect
middle-aged and older, mostly female patients and causes the thyroid to have a firm "woody" texture.
It may be
clinically mistaken for a neck malignancy and can cause symptoms of stridor, dyspnea, dysphasia,
laryngeal
nerve paralysis, or hypothyroidism.

Eyeball protrusion (choice A) suggests the hyperthyroidism of Graves disease.

A massive, soft thyroid gland (choice B) suggests multinodular goiter.

A single large thyroid nodule (choice C) could be due to either a thyroid adenoma or thyroid cancer.

A very tender and painful thyroid (choice D) suggests subacute granulomatous (de Quervain)
thyroiditis.

A 40-year-old woman with polycythemia vera develops progressive severe ascites and tender
hepatomegaly over
a period of several months. Liver function tests are near normal. Which of the following tests would
be most likely
to establish the diagnosis?

A. Endoscopic retrograde cholangiopancreatography (ERCP)

B. Hepatic venography

C. Serum alpha-fetoprotein

D. Serum ceruloplasmin

E. Serum iron studies

Explanation:

The correct answer is B. The clinical presentation is most consistent with Budd-Chiari syndrome
(hepatic vein
obstruction), which may occur as a complication of thrombogenic and myeloproliferative disorders,
including
polycythemia vera. The presentation illustrated is the most common; alternative presentations include
fulminant
liver failure and cases in which intractable abdominal pain is the most prominent initial finding. The
best method
listed to establish the diagnosis of Budd-Chiari syndrome is hepatic venography to demonstrate the
occlusion of
the hepatic venous system. Liver biopsy to provide evidence of centrilobular congestion and
sinusoidal dilatation
14

(in the absence of right-sided heart failure) is definitive, but more invasive.

Endoscopic retrograde cholangiopancreatography (choice A) is most useful in demonstrating lesions


of the
biliary tree.

Serum alpha-fetoprotein (choice C) is a marker for hepatocellular carcinoma.

Ceruloplasmin levels (choice D) are altered in Wilson's disease, in which cirrhosis and brain damage
occur
secondary to abnormalities in the metabolism of copper.

Serum iron studies (choice E) are useful when considering hemochromatosis as a cause of cirrhosis.

A 49-year-old homemaker presents to her physician because she noticed a lump in her breast during
self-examination. Biopsy of the lump demonstrates invasive ductal carcinoma. The connective tissue
adjacent to
the nests of tumor is very densely collagenous. This is an example of which of the following
processes?

A. Anaplasia

B. Carcinoma in situ

C. Desmoplasia

D. Dysplasia

E. Metaplasia

Explanation:

The correct answer is C. This is an example of desmoplasia, which is excessive fibrous tissue
formation in the
stroma of a tumor. The abundant fibrous tissue growth is, itself, benign.

Anaplasia (choice A) is a term used for tumors that show severe loss of cell differentiation and tissue
organization; anaplastic tumors typically are much more clinically aggressive than their well-
differentiated
counterparts.

Dysplasia (choice D) is atypical cellular proliferation (without being so severe as to qualify for the
diagnosis of
cancer); an example is the epithelium seen in tubular adenomas of the colon. In contrast, carcinoma in
situ
(choice B) is a similar change that is severe enough to be classified as cancer, but is confined to the
epithelium
with no invasion of underlying tissue. Carcinoma in situ can be found throughout the epithelial
surfaces of the
body.
15

Metaplasia (choice E) is the replacement of one type of differentiated cell or tissue by another not
normally
present at that site; an example is the replacement, in smokers, of the normal, ciliated, columnar
epithelium of
the respiratory tract with squamous epithelium.

Autopsy of an elderly individual who died in a nursing home with no known genetic diseases reveals small
amounts of amyloid deposition in the heart. Amyloid deposition is not seen in other organs. There is
no history of
long-standing inflammatory disease. This type of amyloid would be most likely to be composed of
which of the
following proteins?

A. Amyloid-associated protein

B. Amyloid light chain protein

C. Beta-2-amyloid protein

D. Beta-2-microglobulin

E. Transthyretin

Explanation:

The correct answer is E. This patient has senile cardiac amyloidosis, which is usually a clinically
insignificant
condition due to deposition of structurally normal transthyretin (formerly called prealbumin). The
transthyretin is
a normal serum protein used to transport thyroxin and retinal. In addition to causing senile cardiac
amyloidosis,
transthyretin, in a mutant rather than normal form, is deposited as amyloid in the familial amyloid
polyneuropathies.

Amyloid-associated protein (choice A) is a more common protein deposited as amyloid, and


precipitates in
secondary amyloidosis associated with underlying chronic inflammatory conditions.

Amyloid light chain protein (choice B) is a common protein deposited as amyloid, and precipitates in
amyloidosis
related to multiple myeloma and other monoclonal B cell proliferations.

Beta-2-amyloid protein (choice C) is deposited as amyloid in the brain (notably in blood vessels and
cerebral
plaques) of patients with Alzheimer's disease.

Beta-2-microglobulin (choice D) is a normal serum protein that is deposited in amyloidosis


complicating
long-term hemodialysis.
16

Routine physical examination of a patient demonstrates proteinuria by dipstick method. No glucose is


detected.
Urine protein electrophoresis demonstrates a monoclonal spike. A tumor of which cell line would
most likely
produce these findings?

A. Plasma cells

B. Renal tubular cells

C. Smooth muscle

D. T-lymphocytes

E. Transitional epithelium

Explanation:

The correct answer is A. The tumor is multiple myeloma, a neoplasm of plasma cells. The
monoclonal spike on
urine protein electrophoresis is typically due to excess light chains (associated with urinary Bence-
Jones
protein).

Monoclonal proteinuria would not be produced by tumors of the the other cell lines listed.

Renal cell carcinoma is a tumor of renal tubular cells (choice B).

Leiomyoma and leiomyosarcoma are tumors of smooth muscle (choice C).

Cutaneous T-cell lymphomas and T-cell leukemia are tumors of T-lymphocytes (choice D).

Transitional cell carcinoma is a tumor of transitional epithelium (choice E).

An 83-year-old female has a biopsy of an ulcerated nipple lesion that is interpreted as Paget's disease. A
biopsy
of the underlying breast tissue will most likely show which of the following?

A. Acute mastitis

B. Ductal carcinoma in situ

C. Intraductal papilloma

D. Invasive lobular carcinoma

E. Normal breast tissue

Explanation:
17

The correct answer is B. Paget's disease of the breast is a form of ductal carcinoma in which
neoplastic cells
involve the squamous epithelium of the skin by direct extension through the lactiferous ducts.
Underlying breast
tissue shows the origin of the ductal carcinoma-usually ductal carcinoma in situ and less frequently
invasive
ductal carcinoma.

Acute mastitis (choice A) is a disease of nursing women in which bacteria gain entry to the breast
tissue via
cracks in the traumatized nipple. It is characterized by acute inflammation and tissue necrosis.

Intraductal papilloma (choice C), a papillary mass arising within the ducts, usually presents as a
single
subareolar tumor that may produce a bloody or serous nipple discharge. Most intraductal papillomas
are
benign and are cured with complete excision.

Invasive lobular carcinoma (choice D) is a tumor of the terminal ductules of the breast. It presents as
a poorly
circumscribed, rubbery breast mass, unlike invasive ductal carcinoma, which tends to appear as a
hard,
stellate, and fibrous tumor. Lobular carcinoma does not produce Paget's disease.

Paget's disease of the breast always reflects underlying duct cancer. This is in marked distinction from
extramammary Paget's disease, which may arise without an identifiable underlying malignancy
(choice E).

A 52-year-old male presents with epigastric pain that improves with meals. Endoscopy demonstrates a 2
cm
ulcerated area located 3 cm distal to the pyloric junction. Basal acid output is within normal limits.
Which of the
following is most likely to have made the strongest contribution to the development of this disease?

A. Aspirin use

B. Chronic antacid use

C. Drinking alcohol

D. Helicobacter pylori infection

E. Smoking

Explanation:

The correct answer is D. The patient has a duodenal peptic ulcer. The strongest risk factor for
duodenal peptic
ulcer is Helicobacter pylori infection, which is found in almost 100% of these cases (contrast to 70%
infection
rate in gastric peptic ulcer). The basal acid output is normal in many patients with duodenal ulcer.
18

Aspirin use (choice A) and ethanol use (choice C) are more strongly implicated in gastric ulcer
disease than
duodenal ulcer disease.

Chronic antacid use (choice B) is seen as a result of peptic ulcer disease, not as a cause of it.

Smoking (choice E) may also be a lesser contributing factor to the development of peptic ulcer.

A 42-year-old woman is noted to have mildly elevated creatinine and blood urea nitrogen on routine
physical
exam. She recalls that her father also had kidney trouble and died in kidney failure. Workup reveals
persistent
azotemia and microscopic hematuria without evidence of urinary tract infection. An ultrasound of the
kidneys
identifies bilaterally enlarged and multicystic kidneys. In addition to chronic renal failure, the
clinician should also
be concerned about her risk of

A. liver failure

B. pancreatic insufficiency

C. portal hypertension

D. renal cell carcinoma

E. subarachnoid hemorrhage

Explanation:

The correct answer is E. Multicystic kidneys, slowly progressive renal failure, and a positive family
history are
characteristics of autosomal dominant (adult) polycystic kidney disease (APKD). This disease
typically presents
in the 40s to 60s and is characterized by marked renal enlargement due to numerous fluid-filled cysts,
which
develop between the normally functioning nephrons. APKD is highly associated with hepatic cysts,
and berry
aneurysms in the circle of Willis that may rupture, producing spontaneous subarachnoid hemorrhage.
Infrequently, APKD also produces cysts in the pancreas, spleen, or lungs, but these are not clinically
relevant.

Hepatic cysts in adult polycystic disease do not ordinarily produce symptoms of hepatic failure
(choice A).

Pancreatic cyst formation in APKD is not generally associated with pancreatic insufficiency (choice
B).

Children with autosomal recessive polycystic kidney disease may develop congenital hepatic fibrosis
with
hypertension and splenomegaly, but this is not part of APKD (choice C).

APKD is not considered a risk factor for renal cell carcinoma or any other type of cancer (choice D).
19

A 32-year-old African American female presents with pelvic pain, low back pain, and a sensation of
"pulling" or
"stretching" in her groin. Bimanual examination reveals a firm mass in the right adnexa. An
ultrasound
examination reveals the presence of fluid in the abdominal cavity and the right thoracic cavity. Which
of the
following conditions is most strongly suggested by this patient's presentation?

A. Ectopic pregnancy

B. Endometrial implant

C. Ovarian fibroma

D. Pelvic inflammatory disease

E. Uterine leiomyoma

Explanation:

The correct answer is C. Meigs syndrome is the unusual combination of hydrothorax (often right
sided), ascites,
and an ovarian tumor (often a benign fibroma). Low back pain and a stretching or pulling sensation
are
characteristic of ascites in some patients (ascites can also be asymptomatic). The etiology of the fluid
accumulation in Meigs syndrome remains a mystery.

Ectopic pregnancy (choice A) would not be associated with hydrothorax. If an ectopic pregnancy
ruptured,
blood could accumulate in the abdominal cavity, but the patient would likely be hypotensive or dead.

An endometrial implant (choice B) could cause pain, or impair fertility, but would not be expected to
produce the
combination of ascites and isolated right-sided hydrothorax.

Pelvic inflammatory disease (choice D) would be unlikely to produce the combination of ascites and
isolated
right-sided hydrothorax, although it might cause low back pain or pelvic pain.

A uterine leiomyoma (choice E) would not be expected to produce an adnexal mass, nor would it be
likely to
cause ascites and hydrothorax.

A 3-year-old child develops severe generalized edema following a viral infection. On the basis of clinical
chemistry tests, a renal biopsy is performed, with normal light microscopic findings. Which of the
following
abnormal laboratory values might be expected in this individual?
20

A. Decreased alpha globulin levels

B. Decreased fibrinogen

C. Increased serum calcium levels

D. Low serum albumin levels

E. Red blood cell casts in the urine

Explanation:

The correct answer is D. This child has minimal change disease, which is the major cause (over 90%
of cases)
of nephrotic syndrome in children aged 2 to 6 years. The most prominent clinical chemistry finding in
these
patients is massive proteinuria. The urinary protein in minimal change disease, in contrast to other
causes of
nephrotic syndrome, is often composed predominantly of albumin. Many other clinical chemistry
changes may
also be seen, including decreased serum albumin levels, hyperlipidemia, increased serum levels of
alpha2- and
beta-globulins, decreased IgG, and increased fibrinogen. Minimal change disease characteristically
shows
normal or near normal appearance of the glomeruli by light microscopy and extensive fusion of foot
processes
of the glomerular podocytes by electron microscopy. A point not always recognized by beginners is
that the
podocyte alterations may represent a reaction to, rather than a cause of, the proteinuria (e.g., an
attempt to
"seal the holes" in the glomerulus), since varying degrees of foot process fusion (together with more
specific
features) may sometimes be seen in other glomerular diseases associated with the nephrotic
syndrome.

Alpha-globulin levels (choice A) would be increased, rather than decreased, in minimal change
disease.

Fibrinogen levels are increased, rather than decreased (choice B).

Serum calcium levels (choice C) are typically decreased in the nephrotic syndrome, possibly due to
renal loss
of vitamin D binding protein.

Red blood cell casts in the urine (choice E) are indicative of glomerulonephritis, rather than the
nephrotic
syndrome.

A 42-year-old African-American man sustains severe injuries in an automobile accident and is admitted to
the
intensive care unit. Examination of a peripheral blood smear on the 3rd day of admission reveals
helmet cells,
21

schistocytes, and decreased platelets. Which of the following is most strongly suggested by these
findings?

A. Autoimmune hemolysis

B. Disseminated intravascular coagulation (DIC)

C. Hereditary spherocytosis

D. Megaloblastic anemia

E. Sickle cell anemia

Explanation:

The correct answer is B. The findings suggest disseminated intravascular coagulation (DIC), which is
a feared
complication of many other disorders, such as obstetrical catastrophes, metastatic cancer, massive
trauma, and
bacterial sepsis. The basic defect in DIC is a coagulopathy characterized by bleeding from mucosal
surfaces,
thrombocytopenia, prolonged PT and PTT, decreased fibrinogen level, and elevated fibrin split
products.
Helmet cells and schistocytes (fragmented red blood cells) are seen on peripheral blood smear.

Autoimmune hemolysis (choice A) and hereditary spherocytosis (choice C) would be characterized


by
spherocytes in the peripheral smear.

Macro-ovalocytes and hypersegmented neutrophils can be seen in megaloblastic anemia (choice D).

Sickle cells are seen in sickle cell anemia (choice E).

A 52-year-old man is found dead in his home. Autopsy reveals hemopericardium secondary to ventricular
wall
rupture. Roughly how long before his death did the man probably have a myocardial infarction?

A. 2 days

B. 7 days

C. 12 days

D. 20 days

E. 60 days

Explanation:

The correct answer is B. Unsuspected (or denied) myocardial infarction is not uncommon, and death
may occur
22

because of untreated complications. A number of serious complications can occur between 5 and 10
days
following infarction, due to marked weakening of the necrotic myocardium. These include rupture of
the
ventricular wall leading to hemopericardium and cardiac tamponade (as this patient had), rupture of
the
interventricular septum, and rupture of the papillary muscle.

Arrhythmias are the most common complication 2 days post-infarction (choice A).

Fibrinous pericarditis secondary to an autoimmune phenomenon (Dressler's syndrome) can be seen


several
weeks after infarctions (choices C and D).

By 60 days after infarction (choice E), the contracted scar is usually complete, and residual
complications
include left ventricular failure and arrhythmias.

A 25-year-old man presents to a rheumatologist with complaints of joint pain involving the large joints of
the legs.
On questioning, the patient indicates that exacerbations in the joint pain are frequently accompanied
by diarrhea.
Which of the following gastrointestinal diseases is most likely to be implicated as the cause of the
patient's joint
problems?

A. Amebic colitis

B. Chronic appendicitis

C. Diverticulosis

D. Pseudomembranous colitis

E. Ulcerative colitis

Explanation:

The correct answer is E. Several gastrointestinal diseases are associated with rheumatologic
complaints. The
most frequent of these are the chronic inflammatory bowel diseases, ulcerative colitis and Crohn's
disease,
which can be associated with sacroiliitis (related to HLA-B27) or lower limb arthritis. Other GI
diseases
associated with arthropathy include bypass surgery, Whipple's disease, Behcet's syndrome, and celiac
disease.

Amebic colitis (choice A) is caused by ingestion of infectious cysts (typically from Entamoeba
histolytica).
Symptoms include abdominal pain and diarrhea; malaise and weight loss may occur. Cecal amebiasis
can
23

resemble acute appendicitis.

Chronic appendicitis (choice B) may be asymptomatic or cause poorly defined abdominal pain.

Diverticulosis (choice C) is usually a disease of older adults. It is often asymptomatic unless


inflammation
supervenes.

Pseudomembranous colitis (choice D) is a severe form of diarrhea usually seen in the setting of prior
antibiotic
use. The causative organism is almost always Clostridium difficile.

A 45-year-old woman complains of difficulty speaking, chewing, and swallowing. She experiences
generalized
weakness that increases with effort and as the day goes on. Symptoms are significantly improved after
taking
neostigmine. Autoantibodies responsible for causing the patient's condition are directed against

A. acetylcholine receptors

B. double-stranded DNA

C. dystrophin

D. erythrocyte surface antigens

E. myelin

Explanation:

The correct answer is A. The patient has myasthenia gravis (MG), which typically produces weakness
worsening over the course of the day. It often affects the eye muscles and can produce diplopia.
Neostigmine,
an acetylcholinesterase inhibitor, would temporarily improve the patient's condition, which is
associated with
antibodies against nicotinic acetylcholine receptors present on skeletal muscle.

Antibodies to double-stranded DNA (choice B) as well as anti-Smith antibodies are found specifically
in systemic
lupus erythematosus. Peripheral nuclear staining is observed on immunofluorescence.

Antibodies to dystrophin (choice C) are not a recognized pathology. Abnormal or absent dystrophin,
resulting
from mutations in the X chromosome, is associated with Becker's and Duchenne muscular dystrophy,
respectively. Pelvic girdle weakness and ataxia are classic symptoms.

Antibodies to erythrocyte surface antigens (choice D) can be found in warm antibody autoimmune
hemolytic
anemia. Patients with this condition would have a positive direct Coomb's test.
24

Antibodies to myelin (choice E) may play a role in multiple sclerosis, which is presumed to be of
autoimmune
etiology. This demyelinating disease is characterized by the spontaneous appearance and remission of
symptoms such as hyperreflexia, weakness, spasticity, dysarthria, tremor, ataxia, and visual
disturbances.
Neostigmine would not produce any improvement.

A 40-year-old woman with polycythemia vera develops progressive severe ascites and tender
hepatomegaly over
a period of several months. Liver function tests are near normal. Which of the following tests would
be most likely
to establish the probable diagnosis?

A. Endoscopic retrograde cholangiopancreatography

B. Hepatic venography

C. Serum alpha fetoprotein

D. Serum ceruloplasmin studies

E. Serum iron studies

Explanation:

The correct answer is B. The clinical presentation is most consistent with Budd-Chiari syndrome
(hepatic vein
obstruction), which may occur as a complication of thrombogenic and myeloproliferative disorders
including
polycythemia vera. The presentation illustrated is the most common; alternative presentations include
fulminant
liver failure and cases in which intractable abdominal pain is the most prominent initial finding.
Hepatic
venography is the best technique of those listed to demonstrate the occlusion of the hepatic venous
system.

Endoscopic retrograde cholangiopancreatography (choice A) is most useful in demonstrating lesions


of the
biliary tree.

Serum alpha fetoprotein (choice C) is a marker for hepatocellular carcinoma.

Ceruloplasmin (choice D) levels are decreased in Wilson's disease.

Serum iron studies (choice E) are useful when considering hemochromatosis as a cause of cirrhosis.

A 10-year-old boy develops an itchy, vesicular rash, which is maximal on his face and trunk. Physical
examination
demonstrates a mixture of lesions, with macules, papules, vesicles, and crusted lesions. The mother
reports that
25

the lesions seem to be occurring in crops. Which of the following is the most likely diagnosis?

A. Herpes simplex I

B. Herpes simplex II

C. Measles

D. Shingles

E. Varicella

Explanation:

The correct answer is E. This is varicella (chicken pox), which is the primary form of infection by the
herpes
zoster (varicella-zoster) virus. Recurrence due to virus harbored in neurons tends to be dermatomal in
distribution and is called shingles. Fever, malaise, headache, and myalgia may also be present,
particularly in
the prodromal phase. Tzanck smear of the base of a vesicle may demonstrate multinucleated giant
cells.
Immunocompromised patients can be treated with acyclovir to prevent dissemination. Chicken pox
may be
complicated by secondary bacterial infection, pneumonia, systemic spread (immunosuppressed
patients),
neurologic involvement (rare), Reye's syndrome (rare), and hemolytic anemia (rare).

Herpes simplex I (choice A) causes oral vesicles and ulcers.

Herpes simplex II (choice B) causes genital vesicles and ulcers.

Measles (choice C) causes a blotchy, nonvesicular rash.

Shingles (choice D) is the recurrent form of herpes zoster infection and usually is localized to a single
dermatome.

Which of the following complications is currently the major limitation to the long-term success of cardiac
transplantation?

A. Allograft rejection

B. Graft arteriosclerosis

C. Graft atherosclerosis

D. Opportunistic infections

E. Lymphoma

Explanation:
26

The correct answer is B. Currently, graft arteriosclerosis (AKA graft vascular disease) is the most
important limit
to the long-term success of heart transplantation. For unknown reasons, the coronary arteries of
transplanted
hearts undergo intimal thickening associated with hyperplasia of myocytes and fibroblasts and
deposition of
matrix. This results in luminal stenosis and myocardial ischemia. Patients may develop myocardial
infarction,
which is clinically silent because the heart is denervated. The overall survival after heart
transplantation is 80%
at 1 year and 60% at 5 years. Do not confuse graft arteriosclerosis with graft atherosclerosis (choice
C).
Atherosclerosis is caused by accumulation of cholesterol esters and development of atheromas.
Atherosclerosis may recur in the coronary arteries of transplanted hearts, but is not a limiting factor in
long-term
success of heart transplantation.

Allograft rejection (choice A) is certainly a major postoperative problem. However, thanks to early
diagnosis
based on periodic endomyocardial biopsy and the availability of immunosuppressant therapy, this
complication
can be prevented or successfully treated.

Although opportunistic infections (choice D) and development of Epstein-Barr related lymphomas


(choice E) are
undesired effects of profound immunosuppression, these complications do not constitute a significant
limitation
to the overall outcome of cardiac transplantation.

A mother takes her 4-year-old to a pediatrician because the child is having chronic, severe headaches.
Physical
examination demonstrates poor visual tracking with one eye, which had not been present 1 year
previously. The
pediatrician orders a CT scan of the head, which demonstrates a cystic 4-cm mass above the pituitary
gland.
Resection of the tumor reveals a cystic lesion filled with dark, oily fluid containing granular debris.
Histologic
examination of the tumor would most likely demonstrate a tumor with areas resembling which of the
following?

A. Autonomic ganglion

B. Brain

C. Skin

D. Thyroid

E. Tooth enamel organ

Explanation:
27

The correct answer is E. The tumor is a craniopharyngioma, which is also called an ameloblastoma
because of
its histologic resemblance to tooth enamel organ, which contains ameloblasts. The resemblance is not
merely
coincidental, because the embryologic development of the pituitary involves both downward growth
from the
brain, forming the posterior lobe of the pituitary, and upward growth from the mouth (from remnants
of Rathke's
pouch), forming the anterior lobe of the pituitary. Craniopharyngiomas may occupy the sella turcica
or may be
found in a suprasellar location, often in the hypothalamus. The tumor may present with mass effects
(as in this
child) or pituitary insufficiency.

Ganglioneuromas contain tissue resembling autonomic ganglia (choice A).

Dermoid cysts (mature teratomas) of the ovary can contain tissue resembling brain (choice B), skin
(choice C),
or thyroid (choice D).

A 67-year-old male develops severe chest pain. He is admitted to the hospital, and diagnosed with a
myocardial
infarction based on his electrocardiogram and serial CK-MB levels. One week later, he again
complains of
precordial pain and develops a fever of 102°F (38.9°C). Physical examination is remarkable for a loud
friction
rub. Which of the following is the most likely diagnosis?

A. Caseous pericarditis

B. Fibrinous pericarditis

C. Hemorrhagic pericarditis

D. Purulent pericarditis

E. Serous pericarditis

Explanation:

The correct answer is B. Different types of pericarditis can be seen in different settings. Fibrinous and
serofibrinous pericarditis may follow acute myocardial infarction (Dressler's syndrome) and can be
seen in
uremia, chest radiation, rheumatic fever, systemic lupus erythematosus, and following chest trauma
(including
chest surgery).

Caseous pericarditis (choice A) is generally due to tuberculosis.

Hemorrhagic pericarditis (choice C) can be seen with tuberculosis, malignant tumors, patients with
bleeding
diatheses, and following chest surgery.
28

Purulent pericarditis (choice D) is seen when pyogenic infections involve the pericardium, e.g., after
cardiothoracic surgery.

Serous pericarditis (choice E) is seen in non-infectious inflammations (rheumatic fever, lupus,


scleroderma,
tumors, uremia).

A 25-year-old man presents with bilateral hearing loss. MRI reveals bilateral tumors within the
cerebellopontine
angles. Surgery is performed, and the tumors are removed. Both are found to be neurilemomas
("schwannoma").
Which of the following is the most likely diagnosis?

A. Metastatic disease

B. Multiple sclerosis

C. Neurofibromatosis type 1

D. Neurofibromatosis type 2

E. Tuberous sclerosis

Explanation:

The correct answer is D. Neurofibromatosis type 2 is an autosomal dominant condition caused by


mutations of a
gene on chromosome 22 coding for a cytoskeleton-related protein called merlin. Much less common
than
neurofibromatosis type 1, it manifests with multiple CNS tumors, the most frequent of which are
schwannomas of
the 8th cranial nerve and meningiomas. Bilateral schwannomas are virtually pathognomonic (ie,
diagnostic) of
neurofibromatosis type 2.

Metastases to the CNS (choice A) are often multiple and usually involve the gray-white matter
junction. Besides
the unusual location, the young age would make this diagnosis highly improbable.

Multiple sclerosis (choice B) is a chronic remitting/relapsing demyelinating disease. It manifests with


focal
neurologic deficits caused by well-circumscribed areas of myelin loss in the white matter of the brain
(usually
periventricular), brainstem, spinal cord, or optic nerves. It is not associated with an increased
incidence of any
type of brain tumor.

Neurofibromatosis type 1 (choice C) is also autosomal dominant and is caused by mutations of a gene
on
29

chromosome 17 coding for neurofibromin, a protein involved in signal transduction. The most
characteristic
clinical features include café-au-lait spots, neurofibromas (tumors of peripheral nerves different from
schwannomas), Lisch nodules (pigmented nodules of the iris), and CNS tumors, but not
schwannomas.

Tuberous sclerosis (choice E), like neurofibromatosis, is a "neurocutaneous syndrome," ie, a


condition
characterized by concomitant neurologic and skin lesions. Tuberous sclerosis is caused by mutations
in two loci,
either TS1 or TS2. Multiple hamartomas of the brain (cortical "tubers") and other organs, shagreen
patches,
ash-leaf patches, and other skin lesions constitute the clinical findings in this disorder.

A 35-year-old man is referred to a psychiatrist because of erratic behavior. The man had been adopted in
infancy, so his family history is not known. Over the next year, he develops uncontrollable erratic
movements,
such that attempts to pick up a cup or use a pencil produce sudden uncontrolled lurches. When he
tries to walk,
he staggers, thrusts, and abruptly changes direction. Eventually, with disease progression, he develops
increasing rigidity and is unable to move, and finally dies ten years after the onset of symptoms.
Which of the
following changes would most likely be seen on examination of his brain at autopsy?

A. Depigmentation of the substantia nigra and locus ceruleus

B. Diffuse cortical atrophy with relative sparing of primary motor and sensory areas

C. Selective frontal and temporal lobe atrophy

D. Striking degeneration of the caudate nucleus

E. Widespread neuronal loss and gliosis in subcortical sites

Explanation:

The correct answer is D. The disease is autosomal dominant Huntington's chorea. The question stem
describes
a typical clinical progression (the family history is usually strikingly positive). Pathological findings
include
severe atrophy of the caudate nucleus (with loss of medium-sized spiny neurons), less severe
involvement of
the putamen and cerebral cortex, and dilation of the lateral ventricles apparent on CT and MRI
studies. The
disorder is known to be caused by expansion of a CAG trinucleotide repeat in a gene on the short arm
of
chromosome 4 coding for a protein called huntingtin. There is no effective therapy.

Choice A is characteristic of Parkinson's disease, characterized by bradykinesia, pill-rolling tremor,


and
cogwheel rigidity.
30

Choice B is characteristic of Alzheimer's disease, a degenerative dementing disorder.

Choice C is characteristic of Pick's disease, a dementing disorder that may be confused with
Alzheimer's
disease. Microscopically, there is gliosis, neuronal loss, and swollen neurons, which may contain
characteristic
Pick bodies (silver-staining cytoplasmic inclusions).

Choice E is characteristic of progressive supranuclear palsy. Patients exhibit an extrapyramidal


syndrome
accompanied by dystonias of the neck and paralysis of downward gaze.

The presence of which of the following features in an atherosclerotic plaque indicates that it has become a
complicated lesion?

A. Cholesterol crystals

B. Chronic inflammatory cells

C. Intimal smooth muscle

D. Lines of Zahn

E. Necrotic cell debris

Explanation:

The correct answer is D. Complicated lesions indicate advanced atherosclerotic disease. They arise in
atherosclerotic plaques, and render them more susceptible to sudden occlusion and acute infarction of
the
supplied tissues. Commonly, the plaque ulcerates or ruptures, and the exposed surfaces, being highly
thrombogenic, precipitate thrombus formation. Thrombi are typified by the lines of Zahn, alternating
layers of
platelets and fibrin (the pale lines) and layers of blood (the dark lines). Beyond thrombus formation,
other
features of a complicated plaque include hemorrhage into the lesion itself, and microembolism by
cholesterol
crystals or calcified debris. Furthermore, the weakened media underlying the plaque may develop an
aneurysmal dilatation. In general, the clinical significance of atherosclerosis is related to the
consequences of
complicated lesions.

The incorrect options all include features of atheromatous plaques, but do not indicate complicated
lesions:

Beneath the endothelium of a plaque there is a fibrous cap composed of smooth muscle (choice C),
chronic
inflammatory cells (choice B) and lipid laden macrophages (foam cells), as well as extracellular
material.

The core of the lesion, which lies between the intima and the media, is composed of necrotic cellular
debris
31

(choice E), with cholesterol crystals (choice A), calcium, and more foam cells.

A 72-year-old female patient with Alzheimer's disease, but no other medical problems, suddenly becomes
comatose and dies due to an intracranial hemorrhage that caused severe damage to her entire left
cerebral
hemisphere. There was no evidence or history of trauma. What is the most likely cause of this
hemorrhage?

A. Epidural hematoma

B. Subdural hematoma

C. Amyloid angiopathy

D. Rupture of berry aneurysm

E. Rupture of Charcot-Bouchard aneurysm

Explanation:

The correct answer is C. Alzheimer's disease patients are prone to large "lobar" hemorrhages that are
usually
centered in the parietal lobe (thus the name "lobar") and may spread to totally destroy an entire
cerebral
hemisphere, resulting in death. This is due to amyloid deposition into the walls of cerebral blood
vessels
(amyloid angiopathy), similar to the amyloid plaques seen in the parenchyma of the brain with this
disease.
Amyloid makes these vessels weak and prone to rupture.

An epidural hematoma (choice A) is a collection of blood above the dura mater, usually due to a blow
to the side
of the head that fractures the temporal bone of the skull and shears the middle meningeal artery. Even
though
this fast-flowing arterial blood usually causes symptoms within the first 24 hours and can cause life-
threatening
mass effects, no traumatic event was involved in this case.

A subdural hematoma (choice B) forms when slow-flowing venous blood collects below the dura
mater due to
leakage from stretched cortical veins as they drain into the superior sagittal sinus. Even though it is
true that
Alzheimer's patients are more susceptible to these bleeds due to brain atrophy, causing cortical veins
to be
maximally stretched and prone to tearing, subdural hematomas are usually associated with some sort
of trauma.
Furthermore, a subdural hematoma causes gradual symptoms over time, rather than presenting as a
sudden
devastating event as in this case.

Rupture of a berry aneurysm (choice D) usually causes a subarachnoid hemorrhage, in which blood
leaks into
32

the space between the arachnoid membrane and the brain. The blood may also gain access to the
ventricles,
but does not generally damage the cerebral hemispheres. Subarachnoid hemorrhages can be sudden
and
deadly, but usually present with a severe headache ("the worst headache of my life"). These
aneurysms are not
especially associated with Alzheimer's disease.

Rupture of a Charcot-Bouchard aneurysm (choice E) might cause a sudden hemorrhage within the
brain
causing coma and death, but it is not especially associated with Alzheimer's disease. Charcot-
Bouchard
aneurysms are primarily seen in patients with severe hypertension.

A 37-year-old woman presents to her physician complaining of difficulty reading and fatigue. She reports
having a
"pins and needles" feeling in her left arm several months ago that resolved without treatment. On
examination,
visual field deficits and mild hyperreflexia are noted. MRI confirms the suspected diagnosis. Which of
the following
is the underlying mechanism of this patient's disease?

A. Antibodies to acetylcholine receptors

B. Axonal degeneration

C. Demyelination of the peripheral nerves

D. Loss of oligodendrocytes

E. Loss of Schwann cells

Explanation:

The correct answer is D. This woman presents with the classic signs and symptoms of multiple
sclerosis (MS). A
key to this disease is different neurological signs that are separated by space and time. (Another
classic clue
might have been oligoclonal bands on electrophoresis of the CSF.) MS is a demyelinating disease of
the central
nervous system, characterized by loss of oligodendroglial cells, which are the cells that are
responsible for
producing myelin in the central nervous system. Diagnosis can be confirmed by an MRI revealing
sharply
delineated regions of demyelination (plaques) throughout the central nervous system white matter
(especially in
periventricular areas).

Antibodies to acetylcholine receptors (choice A) have been implicated in the etiology of myasthenia
gravis, not
multiple sclerosis.
33

MS is generally characterized by axonal preservation, rather than degeneration (choice B).

Demyelination of peripheral nerves (choice C) occurs in a number of diseases (e.g., Guillain-Barré),


but not in
MS. Guillain-Barré is characterized by ascending muscle weakness, areflexia, and paralysis.

Oligodendrocytes are responsible for producing myelin in the central nervous system; Schwann cells
(choice E)
are responsible for myelination in the peripheral nervous system, and are not affected in multiple
sclerosis.

Which of the following endometrial lesions is associated with the highest risk of developing endometrial
adenocarcinoma?

A. Chronic endometritis

B. Complex hyperplasia with atypia

C. Complex hyperplasia without atypia

D. Simple hyperplasia

E. Squamous metaplasia

Explanation:

The correct answer is B. In general, any condition characterized by excessive estrogenic stimulation is
associated with some degree of endometrial hyperplasia and increased risk of endometrial cancer.
Endometrial
hyperplasia is a histologic precursor of endometrial adenocarcinoma. Hyperplasia may progress to
invasive
adenocarcinoma through progressive degrees of cellular and architectural atypia. The grade of
hyperplasia,
therefore, is related to the severity of alterations of gland architecture, growth pattern, and cytologic
features.
The most severe changes are present in complex hyperplasia with atypia. Disorganization and
crowding of
glands, high mitotic activity, and nuclear atypia characterize this change. Longitudinal studies show
that 25% of
women with this form of hyperplasia develop adenocarcinoma. Complex hyperplasia without atypia
(choice C) is
characterized by similar crowding of glands and epithelial cells in the absence of cellular atypia:
progression to
cancer occurs in 5% of cases. In simple hyperplasia (choice D), the glands are dilated and irregular,
but
cytologic atypia and mitoses are absent and transformation is rare.

Chronic endometritis (choice A) has no relationship with endometrial adenocarcinoma and refers to a
condition
of chronic inflammatory infiltration of the endometrium. A diagnosis of chronic endometritis is made
only when
34

plasma cells are found on biopsy. (Lymphocytes are normally present in the endometrial mucosa.)
This
condition is associated with pelvic inflammatory disease (PID), intrauterine contraceptive devices
(IUD), and
retention of gestational tissue within the uterine cavity.

Squamous metaplasia (choice E) is occasionally found in endometrial biopsy and results from
transformation of
the normal columnar epithelium into squamous epithelium. It is not associated with endometrial
adenocarcinoma.

A patient comes to medical attention after he sets off the metal detector at the airport, despite removing his
watch, belt buckle, and every other obvious source of metal. Which of the following diseases might be
responsible
for this phenomenon?

A. Argyria

B. Gall stones

C. Hemochromatosis

D. Kidney stones

E. Wilson's disease

Explanation:

The correct answer is C. Hemochromatosis is an iron storage disorder that can cause cirrhosis (with
increased
risk of hepatocellular carcinoma), skin pigmentation, pancreatic damage leading to diabetes mellitus,
and
congestive heart failure. These complications are due to damage caused by deposition of iron in
tissues; the
total body iron in some of these individuals may reach 50 g, large enough to set off some airport
metal
detectors.

Argyria (choice A) is a blue-gray skin discoloration related to silver poisoning.

Neither gallstones (choice B) nor kidney stones (choice D) contain metal.

In Wilson's disease (choice E), copper is deposited in liver and brain, but this would not be detected
by metal
detectors.

A Pap smear of a 23-year-old woman demonstrates squamous cells with enlarged, hyperchromatic nuclei
and
prominent perinuclear halos. The Pap smear is graded as cervical intraepithelial neoplasia, grade II
(CIN II).
Which of the following viruses is most likely to be etiologically related to this neoplastic growth?
35

A. Epstein-Barr virus (EBV)

B. Hepatitis B virus (HBV)

C. Human herpesvirus 8 (HHV8)

D. Human papillomavirus (HPV)

E. Human T-cell lymphotropic virus -1 (HTLV-1)

Explanation:

The correct answer is D. CIN II corresponds to moderate dysplasia of the cervix. The CIN lesions of
all grades
(including condyloma) and the cervical cancers that can arise from them appear to be associated with
infection
with certain subtypes of human papilloma virus (HPV). Koilocytotic atypia (enlarged,
hyperchromatic nuclei and
prominent perinuclear halos) is commonly observed with HPV infection. HPV is usually spread
through sexual
contact and also causes penile and anal condyloma and carcinomas.

EBV (choice A) is associated with nasopharyngeal carcinoma and Burkitt's lymphoma.

HBV (choice B) is associated with hepatocellular carcinoma.

HHV8 (choice C) is associated with Kaposi's sarcoma.

HTLV-1 (choice E) is associated with adult T-cell leukemia.

A young man presents to his physician's office for a physical exam. He is concerned because his father died
of a
heart attack in his late 40's. The physician finds that he has elevated serum cholesterol and LDL levels,
but his
VLDL and triglycerides are normal. Further investigation reveals an LDL receptor deficiency. This
patient has
which of the following types of hyperlipidemia?

A. Type I

B. Type IIa

C. Type IIb

D. Type III

E. Type IV

F. Type V
36

Explanation:

The correct answer is B. There are many clues in the question that should have guided you to this
choice. The
laboratory findings are classic for Type IIa hyperlipidemia. These patients have LDL receptor
deficiencies and
are at a great risk of advanced coronary atherosclerosis. Since it is autosomal dominant, the patient's
father
could have been affected as well.

Type I hyperlipidemia (choice A), or familial hyperchylomicronemia, is caused by a lipoprotein


lipase deficiency.
These patients have high serum triglycerides and normal cholesterol. They do not have a substantially
higher
risk of atherosclerosis.

Type IIb hyperlipidemia (choice C), or familial combined hyperlipidemia, presents as elevated serum
LDL, VLDL,
cholesterol, and triglycerides. These patients do have an increased incidence of atherosclerosis.

Type III hyperlipidemia (choice D), or familial dysbetalipoproteinemia, presents as increased serum
cholesterol
and triglycerides. The mode of inheritance is not understood, but apoprotein E is affected and the risk
of
atherosclerosis is great.

Type IV hyperlipidemia (choice E), or familial hypertriglyceridemia, presents as increased


triglycerides with
normal cholesterol and LDL. The disease may be sporadic and is possibly associated with an
increased risk for
atherosclerosis.

Type V hyperlipidemia (choice F), or mixed hypertriglyceridemia, is not common. Cholesterol is


slightly
increased and triglycerides are greatly increased. There is deficient apoprotein CII. The risk of
atherosclerosis
is not clear.

A pregnant woman develops deep, boring pain of her left thigh muscles associated with swelling and
enhanced
warmth of the same leg. The pain is worsened by extending the foot. The superficial veins of the leg
are
engorged. Her condition puts her at risk for which of the following?

A. Acute renal failure

B. Cerebral hemorrhage

C. Hepatic infarction

D. Myocardial infarction
37

E. Pulmonary embolus

Explanation:

The correct answer is E. The patient has clinical findings strongly suggestive of deep venous
thrombosis (DVT).
Pain that increases upon extension of the foot is referred to as Homans' sign. Pregnancy, particularly
during the
third trimester, induces a hypercoagulable blood state (possibly to limit the chance of fatal
hemorrhage during
delivery), which can manifest as DVT. The major complication of DVT is pulmonary embolism,
which may be
massive and can cause sudden death.

Renal failure (choice A) can be a serious problem in pregnancy, but is not related to deep vein
thrombosis.

Cerebral hemorrhage (choice B) and hepatic infarction (choice C) are complications of preeclampsia.

The hypercoagulability that predisposes for deep vein thrombosis also predisposes for myocardial
infarction
(choice D), but MI is not considered a complication of DVT.

A 31-year-old African-American female dancer injures herself during a dance routine and is seen in the
emergency room for tender ribs. A chest radiograph is performed which shows no rib fractures, but
does reveal
significant bilateral hilar adenopathy and middle and upper zone linear streaks and nodules. Upon
further
questioning she recalls an episode approximately one year ago, in which she experienced fever, ankle
swelling,
and tender red bumps on her lower extremities. She currently has no medical complaints and is able to
rehearse
without any difficulty breathing. Which of the following is the most likely diagnosis?

A. No disease, the patient is healthy

B. Pneumocystis carinii pneumonia

C. Sarcoidosis

D. Systemic sclerosis

E. Wegener's granulomatosis

Explanation:

The correct answer is C. This patient has sarcoidosis, a multi-system disease of unknown etiology
characterized by noncaseating granulomas on histological examination of various organs. Sarcoidosis
occurs
38

mostly in blacks in the third to fifth decades. Approximately 90% of cases involve the lungs, with
findings such
as hilar adenopathy and pulmonary infiltration. Other findings include uveitis, erythema nodosum (the
lesions
on this patient's lower extremities), arthritis, central and peripheral neuropathies, cardiomyopathy, and
hypercalcemia. Symptoms include fatigue, exertional dyspnea, and non-productive cough. Diagnosis
is made
after excluding other causes by examination of a tissue biopsy. Radiographic staging is performed by
serial
chest x-rays.

Pneumocystis carinii pneumonia (PCP) (choice B) is a disease almost exclusively of HIV-infected


patients,
usually with a characteristic chest radiograph appearance of bilateral fluffy infiltrates.

Systemic sclerosis (choice D), or scleroderma, is a systemic collagen-vascular disease that involves
the skin
nearly 100% of the time, and the esophagus in approximately 75% of patients.

Wegener's granulomatosis (choice E) is a disorder characterized by focal necrotizing vasculitis in the


lung and
upper airways associated with granuloma formation, and necrotizing glomerulitis.

Following a respiratory infection, a 20-year-old man goes to his physician for a follow-up visit. Physical
examination is unremarkable, but dipstick analysis of his urine reveals marked proteinuria and
microscopic
hematuria. The young man is referred to a specialist, who performs a renal biopsy.
Immunofluorescence
microscopy of the biopsy tissue demonstrates IgA deposition in the glomerular mesangium. These
results are
most consistent with which of the following disorders?

A. Berger's disease

B. Goodpasture's syndrome

C. Minimal change disease

D. Poststreptococcal glomerulonephritis

E. Systemic lupus erythematosus

Explanation:

The correct answer is A. Berger's disease, or Ig A nephropathy, may develop after a respiratory
infection. It is a
major cause of recurrent microscopic hematuria, and may progress to renal failure in a number of
cases. (Note:
don't confuse Berger's disease with Buerger's disease-a vasculitis that occurs in smokers.)

Questions about Goodpasture's syndrome (choice B) often contain a clue about linear deposition of
IgG
anti-basement membrane antibodies.
39

You should associate fusion of podocyte foot processes with minimal change disease (choice C).

Classic poststreptococcal glomerulonephritis (choice D) may follow pharyngitis caused by group A


streptococcus. This type of glomerulonephritis is associated with granular immunofluorescence and
subepithelial humps by electron microscopy.

Renal involvement in lupus (choice E) can have many manifestations, but there will usually be
evidence of a
systemic inflammatory disease.

A 63-year-old African-American male presents with vague complaints of abdominal and back pain,
malaise,
nausea, and weakness, which have been present for 3 or 4 months. Review of systems reveals a 15
pound
weight loss, occasional vomiting, and several episodes of unilateral leg swelling, which have involved
both legs at
different times. These findings are most consistent with which of the following diagnoses?

A. Pancreatic cancer

B. Primary sclerosing cholangitis

C. Pyelonephritis

D. Reflux esophagitis

E. Splenic infarction

Explanation:

The correct answer is A. Pancreatic carcinoma often presents with vague abdominal, back, and
gastrointestinal
complaints; and physical examination is generally unrevealing. The tremendous weight loss is very
suspicious
for carcinoma, and the migrating thrombophlebitis (Trousseau's sign) is extremely helpful in making
the
diagnosis, which should be confirmed with ultrasonography or CT. Although any carcinoma can elicit
migratory
thrombophlebitis, it is mostly associated with tumors of the pancreas, lung, and colon.

Primary sclerosing cholangitis (choice B) is an inflammatory fibrosing disease of the biliary tree of
unknown
etiology, although highly associated with inflammatory bowel disease. It presents with symptoms of
liver failure
(jaundice, pruritus) and progresses to biliary cirrhosis.

Pyelonephritis (choice C) may present with back pain that usually localizes to the costovertebral
angle, and is
generally associated with fevers and dysuria. Chronic pyelonephritis proceeds to hypertension and
renal
40

failure.

Reflux esophagitis (choice D) presents with heartburn, regurgitation, and dysphagia and may
occasionally be
confused with a heart attack by the patient. Long term consequences include bleeding, strictures, and
Barrett's
esophagus.

Splenic infarction (choice E), typically associated with arterial thromboembolic events, generally is
an
asymptomatic occurrence that does not produce clinical symptoms unless the entire spleen is lost.

A 37-year-old man presents to a physician because of a lesion on the shaft of his penis. On physical
examination, a solitary, thickened, whitish plaque with a slightly ulcerated, crusted surface is
observed. Biopsy
reveals markedly dysplastic epithelial cells, many mitoses (some of which are abnormal), disordered
epithelial
maturation, and an intact basement membrane with no evidence of stromal invasion. Which of the
following is the
most likely diagnosis?

A. Bowenoid papulosis

B. Bowen's disease

C. Condyloma acuminatum

D. Erythroplasia of Queyrat

E. Squamous cell carcinoma

Explanation:

The correct answer is B. This is Bowen's disease, which is one clinical form of penile carcinoma in
situ (the
other forms are Bowenoid papulosis and erythroplasia of Queyrat). Untreated Bowen's disease will,
over a
period of years, progress to frank invasive carcinoma in 10% to 20% of patients. Bowen's disease can
affect
the genital region of both men and women.

Bowenoid papulosis (choice A) is another form of penile carcinoma in situ, characterized clinically
by multiple
reddish-brown papular lesions.

Condyloma acuminatum (choice C) usually produces a papillary lesion.

Erythroplasia of Queyrat (choice D) is another form of penile carcinoma in situ, characterized by


single or
multiple shiny red plaques.

Squamous cell carcinoma (choice E) will be frankly invasive on biopsy.

A physician has been treating a 60-year-old patient with renal failure due to polycystic kidney disease. The
41

physician should be specifically concerned about the possible coexistence of which of the following
conditions?

A. Aneurysm of aortic root

B. Atherosclerotic aneurysm

C. Berry aneurysm

D. Cystic medial necrosis

E. Mycotic aneurysm

Explanation:

The correct answer is C. There is a specific association between the adult form of polycystic kidney
disease
and congenital berry aneurysms of the circle of Willis. These aneurysms can rupture, producing a
subarachnoid hemorrhage, and possibly causing death.

Aneurysms of the aortic root (choice A) are associated with syphilis.

Atherosclerosis (choice B) causes abdominal aortic aneurysms, many of which are seen in diabetics.

Cystic medial necrosis (choice D) can cause dissecting aneurysms (e.g., in Marfan's syndrome).

"Mycotic" aneurysms (choice E) are due to bacterial, not fungal, infection.

A 50-year-old woman with a swan-neck deformity of the hands and enlarged knuckles develops large
subcutaneous nodules near her elbows. If the nodules were biopsied, which of the following best
describes their
likely histological appearance?

A. Amorphous crystalline mass surrounded by macrophages

B. Cystic space caused by myxoid degeneration of connective tissue

C. Darkly pigmented synovium with an exuberant, villous growth

D. Fibrinoid necrosis surrounded by palisading epithelioid cells

E. Well-encapsulated nodule of polygonal cells within a tendon sheath

Explanation:

The correct answer is D. Swan-neck deformity, enlarged knuckles, and subcutaneous nodules are
classic clues
42

for rheumatoid arthritis. The subcutaneous rheumatoid nodules are composed histologically of areas
of fibrinoid
necrosis surrounded by palisading epithelioid cells.

Gout tophi are amorphous crystalline masses surrounded by macrophages (choice A).

Ganglion cysts are small cystic spaces caused by myxoid degeneration of connective tissue (choice
B).

Pigmented villonodular synovitis causes darkly pigmented synovium with exuberant villous growth
(choice C).

Nodular tenosynovitis causes a well-encapsulated nodule of polygonal cells within a tendon sheath
(choice E).

A 21-year-old female with a several month history of easy bruising and increased menstrual flow is
evaluated for
a bleeding disorder. Her platelet count is 31,000/mm3. Subsequent investigations determine that she
has
idiopathic thrombocytopenic purpura (ITP). In this disorder, the low platelet count is due to which of
the following?

A. Antiplatelet antibodies

B. Defective platelet aggregation

C. Hypersplenism

D. Ineffective megakaryopoiesis

E. Mechanical trauma

Explanation:

The correct answer is A. ITP is a chronic autoimmune disorder in which antibodies against platelet
glycoproteins
cause platelet destruction and removal by the reticuloendothelial system. Secondary
thrombocytopenia can also
be produced by lupus, viral infections, and drugs. Only when secondary thrombocytopenia has been
ruled out,
can the diagnosis of ITP be made.

Defective platelet aggregation (choice B) is responsible for thrombasthenia, an autosomal dominant


disease
that causes prolonged bleeding time but normal numbers of platelets.

Hypersplenism (choice C) causes thrombocytopenia when an enlarged spleen traps normal platelets in
the
absence of other specific platelet disorders. This type of thrombocytopenia can be cured with
splenectomy.
Although the thrombocytopenia in ITP often improves with splenectomy, ITP does not cause
splenomegaly.
43

Megakaryopoiesis (choice D) is disturbed in any disorder that causes bone marrow failure, including
drug
toxicity, leukemia, and infections. In this setting, thrombocytopenia is often part of a pancytopenia.

Mechanical trauma (choice E) causes thrombocytopenia in disseminated intravascular coagulation


(DIC),
thrombotic microangiopathies (thrombotic thrombocytopenic purpura and hemolytic-uremic
syndrome), and with
giant hemangiomas.

A patient with prolonged diarrhea undergoes esophagogastroduodenoscopy. Biopsy of the small intestine
demonstrates numerous crescent-shaped protozoa adjacent to the epithelial brush border. Which of the
following
organisms is the most likely pathogen?

A. Entamoeba histolytica

B. Escherichia coli

C. Giardia lamblia

D. Naegleria fowleri

E. Trichomonas vaginalis

Explanation:

The correct answer is C. The probable organism is Giardia lamblia, which characteristically infects the
small
intestine. When seen in smears from duodenal aspirates, this flagellated organism has a characteristic
"face-like" appearance. However, in biopsy specimens, the organism is often caught at an angle, and
the
characteristic appearance and location is as described in the question stem. Giardia is a common
contaminant
of water supplies (even in the United States), and patients who ingest the cysts may be asymptomatic
or may
occasionally develop prolonged diarrhea or intestinal malabsorption. Some patients with severe
disease have
low serum IgA or low overall immunoglobulin levels.

Entamoeba histolytica (choice A) usually infects the large intestine and/or liver.

Escherichia coli (choice B) is a bacterial cause of diarrhea.

Naegleria fowleri (choice D) causes meningoencephalitis.

Trichomonas vaginalis (choice E) causes vaginitis.


44

A 60-year-old man presents to the emergency room with severe, tearing pain radiating to the upper back.
Over a
period of hours the pain moves to the mid back and then involves both flanks. Hematuria develops
shortly
thereafter. Which of the following renal complications has most likely occurred?

A. Acute glomerulonephritis

B. Bilateral renal infarction

C. Polycystic kidney disease

D. Pyelonephritis

E. Sickle cell crisis

Explanation:

The correct answer is B. The patient has a dissecting aortic aneurysm that occluded the renal arteries,
causing
bilateral renal infarction with flank pain and hematuria. This is a very dangerous complication of
dissecting aortic
aneurysm, which classically presents as described in the question stem.

Acute glomerulonephritis (choice A) is characterized by hematuria, red cell casts, and often,
proteinuria and
edema. This syndrome typically develops over days, not hours, and would not be expected to result
from
dissecting aortic aneurysm.

Polycystic kidney disease (choice C) is a lifelong condition characterized by bilateral polycystic


changes in the
kidneys. It is associated with hypertension and eventual renal failure.

Pyelonephritis (choice D) is an infection of the kidney that can develop as a complication of bacterial
endocarditis, or as a consequence of vesicoureteral reflux, but would not be expected to result from
aortic
dissection.

Sickle cell crisis (choice E) can cause papillary necrosis of the kidneys with hematuria, but there is no
indication
that the patient has sickle cell anemia.

During a routine preemployment physical examination, an apparently healthy 24-year-old man is found to
have
increased serum levels of unconjugated bilirubin. Conjugated bilirubin and transaminases are normal.
Careful
questioning reveals no evidence of recent illnesses. Which of the following is the most likely
diagnosis?
45

A. Crigler-Najjar syndrome

B. Dubin-Johnson syndrome

C. Gilbert's syndrome

D. Rotor syndrome

E. Wilson's disease

Explanation:

The correct answer is C. Gilbert's syndrome is a relatively common hereditary condition that is so
mild, a case
can be made for classifying it as a normal variant rather than a true disease. It produces asymptomatic
unconjugated hyperbilirubinemia, and is important to know about to avoid unnecessary diagnostic
procedures.

Crigler-Najjar syndrome (choice A) represents several hereditary diseases characterized by severe


unconjugated hyperbilirubinemia. The type I form, in particular, can be fatal.

Dubin-Johnson (choice B) and Rotor (choice D) syndromes are relatively mild hereditary conjugated
hyperbilirubinemias.

Wilson's disease (choice E) is a serious disorder of copper metabolism associated with hepatic
cirrhosis,
movement disorder, and pathognomonic gold-colored rings (Kayser-Fleischer rings) in the iris.
Diagnosis is
based on reduced serum ceruloplasmin (a copper-binding protein), increased hepatic copper content,
and
increased urinary copper excretion.

Which of the following CNS tumors has the worst prognosis and is associated with the shortest survival?

A. Anaplastic astrocytoma (WHO grade III)

B. Glioblastoma multiforme

C. Meningioma

D. Oligodendroglioma

E. Well-differentiated astrocytoma (WHO grade II)

Explanation:

The correct answer is B. Glioblastoma multiforme (GBM) is the most common and most malignant of
the
primary CNS neoplasms. GBM belongs to the category of gliomas called astrocytomas, which
originate from
neoplastic transformation of astrocytes. There is a continuum in the anaplastic features of
astrocytomas, from
46

well-differentiated astrocytoma, characterized by minimal atypia and mitotic activity, to GBM,


characterized by a
brisk mitotic rate, multifocal necrosis, and extreme nuclear pleomorphism. Since histologic atypia has
been
shown to correlate with biologic behavior, several grading systems have been developed to help
predict the
prognosis of astrocytomas. The most common of such grading systems (WHO and Saint Anne-Mayo)
divide
astrocytomas into four grades, from grade I (benign, well-differentiated) to grade IV (malignant,
poorly
differentiated). GBM is grade IV astrocytoma. It is associated with a poor prognosis; most patients
die within
approximately 12 months of diagnosis.

Well-differentiated astrocytoma (choice E) is basically a synonym for grade II astrocytoma, while


anaplastic
astrocytoma (choice A) refers to grade III astrocytoma. Survival is better than GBM, although
definitive cure is
rare, even for well-differentiated astrocytomas.

Meningioma (choice C) is a benign tumor of meningothelial origin that grows from the dura mater,
pushing,
rather than infiltrating, the underlying brain. Nevertheless, meningiomas may infrequently show
histologic
features of malignancy and infiltrate the brain, in which case the prognosis is worse (though still
better than the
dreadful GBM).

Oligodendroglioma (choice D) arises from oligodendroglial cells and usually lacks histologic markers
of
malignancy (e.g., mitotic activity, nuclear atypia, and necrosis). Owing to its infiltrative pattern of
growth,
however, complete excision is virtually impossible (remember, there is no basement membrane or
other
anatomic boundary in the CNS to contain the spread of a glial neoplasm). Thus, oligodendrogliomas
recur over
and over following each surgical resection. After each recurrence, these tumors acquire progressively
increasing degrees of anaplasia, ultimately becoming similar to GBM. Survival can be very long (7-
10 years),
but definitive cure is exceptional.

Following penicillin treatment for pneumococcal pneumonia, a patient develops a palpable purpuric rash.
Biopsy of
the rash demonstrates vasculitis with hemorrhage into the skin. The involved arterioles and venules
show fibrinoid
necrosis and a neutrophilic infiltrate into the wall. Many of the neutrophils are fragmented. Which of
the following is
the most likely diagnosis?

A. Allergic granulomatosis and angiitis

B. Giant cell arteritis


47

C. Leukocytoclastic angiitis

D. Polyarteritis nodosa

E. Wegener's granulomatosus

Explanation:

The correct answer is C. The correct diagnosis is leukocytoclastic angiitis, which is also known as
hypersensitivity angiitis or &ldquo;microscopic&rdquo; polyarteritis nodosa. Affected vessels are
usually smaller
(arterioles, venules, and capillaries) than those of classic polyarteritis nodosa, and consequently,
infarction is
less common. The microscopic appearance is that described in the question stem; the fragmentation of
the
neutrophils is &ldquo;leukocytoclasis.&rdquo; Cases can be either limited to the skin or generalized.
Specific
antigens (e.g., penicillin) may be suspected as possible triggers, leading to the alternative term
&ldquo;hypersensitivity angiitis.&rdquo; Removal of the offending antigen in these cases may induce
clinical
improvement.

Allergic granulomatosis and angiitis (choice A) is an alternative name for Churg- Strauss disease,
which affects
the respiratory and renal systems.

Giant cell arteritis (choice B) is an alternative name for temporal arteritis, which affects the temporal
and
ophthalmic arteries.

Polyarteritis nodosa (choice D) primarily affects vessels larger than arterioles.

Wegener's granulomatosus (choice E) affects the respiratory and renal systems.

A 60-year-old man suddenly becomes completely blind in one eye, and angiography demonstrates
occlusion of
the central retinal artery. Which of the following is the most likely cause of the occlusion?

A. Atheroma or embolism

B. Cranial (temporal) arteritis

C. Hypertension

D. Polycythemia vera

E. Tumor

Explanation:
48

The correct answer is A. The point of this question is that sometimes the obvious explanation is the
correct one.
Occlusion of the central retinal artery rapidly causes irreversible blindness with loss of the inner
retinal layers.
(The photoreceptor rod and cone cells are maintained by the pigment epithelium.) The site of
occlusion is
typically just posterior to the cribriform plate. A garden-variety atheroma or embolism is
overwhelmingly the
most common cause of central retinal artery occlusion.

Despite all of the teaching about the risk of blindness in temporal arteritis (choice B), this disorder
causes only
10% of central retinal artery occlusions.

Hypertension (choice C) is more apt to cause bleeding than thrombosis.

Polycythemia vera (choice D) could (rarely) cause occlusion because of increased blood viscosity and
a
tendency for thrombosis.

Tumor (choice E) might also cause retinal artery thrombosis, but this would be far rarer than
atheroma.

During a pre-employment physical, a 45-year-old man is noted to have a 3 cm palpable nodule in one lobe
of an
otherwise normal sized thyroid gland. Needle aspiration of the nodule demonstrates polygonal tumor
cells and
amyloid, but only very scanty colloid and normal follicular cells. Which of the following is the most
likely diagnosis?

A. Follicular thyroid carcinoma

B. Hashimoto's disease

C. Medullary thyroid carcinoma

D. Papillary thyroid carcinoma

E. Thyroid adenoma

Explanation:

The correct answer is C. The most distinctive clue is the presence of amyloid, which specifically
suggests
medullary thyroid carcinoma. This carcinoma is a tumor of the neuroendocrine parafollicular cells of
the thyroid,
rather than the follicular lining epithelium. These cells produce calcitonin, the precursor protein of
which can
precipitate, forming bands and nodules of amyloid that appear histologically identical to other forms
of amyloid.
The other thing to remember about medullary carcinoma of the thyroid is that it can be a component
of multiple
49

endocrine neoplasia (MEN) syndromes type IIa (parathyroid disease, pheochromocytomas, medullary
carcinoma) and type IIb (parathyroid disease, pheochromocytomas, medullary carcinoma,
mucocutaneous
ganglioneuromas, Marfanoid habitus).

Because follicular thyroid carcinoma (choice A) closely resembles normal thyroid tissue, it usually
cannot be
reliably diagnosed based on fine needle aspiration alone.

Hashimoto's disease (choice B) would show lymphocytes, plasma cells, and macrophages on
aspiration.

Papillary thyroid carcinoma (choice D) can be diagnosed by aspiration if papillary clusters are seen.

Thyroid adenoma (choice E) cannot be reliably distinguished from well-differentiated thyroid


carcinoma on
aspiration.

A 56-year-old alcoholic man is brought in to the emergency room after being found unconscious by his
daughter,
who called the paramedics. Paramedics report finding the man in a stuporous condition in the bathtub,
covered
with vomit. On arrival to the emergency room, the man is clammy and his blood pressure is 85/50.
Which of the
following conditions is the most likely cause of his hypotension?

A. Acute hemorrhagic pancreatitis

B. Chronic calcifying pancreatitis

C. Chronic obstructive pancreatitis

D. Cystic fibrosis

E. Pancreatic pseudocyst

Explanation:

The correct answer is A. Acute hemorrhagic pancreatitis is a life-threatening abdominal emergency


that is most
often seen in the setting of excessive acute alcohol or food ingestion. In this condition, activated
pancreatic
enzymes are released into the tissues, where they cause severe local damage to the pancreas, with pain
radiating to the back. The enzymes are also released into the blood stream. Shock may result from
hemorrhage, activation of bradykinin and related peptides, and/or release of proteolytic and lipolytic
enzymes
into the circulation. Other systemic manifestations include hypocalcemia, glucose intolerance, and
jaundice.

Chronic calcifying pancreatitis (choice B) is seen in chronic alcoholics, but does not cause the
dramatic
presentation of acute hemorrhagic pancreatitis.
50

Chronic obstructive pancreatitis (choice C) is seen in gallstone disease.

Cystic fibrosis (choice D) is an inherited disease that usually causes death by age 30.

Pancreatic pseudocyst (choice E) is an acquired loculation of fluid that may be seen after pancreatitis
or
trauma.

An abdominal x-ray performed on a 54-year-old man demonstrates a large, irregular, calcified mass with
multiple
broad projections filling one renal pelvis. Which of the following laboratory findings might be
expected in this
patient?

A. Decreased urine pH

B. Hypercalcemia

C. Hyperuricemia

D. Increased ammonia concentration in the urine

E. Increased cystine concentration in the urine

Explanation:

The correct answer is D. The patient has a stag-horn calculus. These very large calculi are almost
always
composed principally of magnesium ammonium phosphate (often with enough calcium to be radio-
opaque) and
form in the setting of infection by urea-splitting bacteria such as Proteus.

Increased urine ammonia concentrations are a byproduct of the bacterial metabolism of urea, and tend
to
increase urine pH (compare with choice A).

Hypercalciuria, with or without hypercalcemia (choice B), is a cause of calcium oxalate stones.

Uric acid stones can be seen in patients with hyperuricemia (choice C) secondary to gout, or in
conditions in
which a very rapid cell turnover occurs (e.g., leukemias).

Genetically determined defects in the renal transport of amino acids are associated with cystine stones
(choice
E).

Oral examination of a 57-year-old female reveals a 1-cm, flat, white patch on the buccal mucosa. Which of
the
51

following diagnoses indicates the greatest likelihood that this lesion will progress to an oral
malignancy?

A. Hairy leukoplakia

B. Leukoplakia

C. Lichen planus

D. Oral thrush

E. Squamous papilloma

Explanation:

The correct answer is B. Leukoplakia is a white plaque on the oral mucosa for which a more specific
diagnosis
cannot be rendered. Leukoplakia is often associated with hyperkeratosis and may or may not show
dysplastic
squamous epithelium. On average, 5% of leukoplakias contain in situ or overt carcinoma.

Hairy leukoplakia (choice A) is an oral infective lesion seen almost exclusively in HIV infection. It is
a fluffy, white
hyperkeratotic lesion in which a destructive piling up of keratotic squames is seen. Hairy leukoplakia
is
associated with viral infection, mostly EBV, HPV, and/or HIV. It does not progress to cancer.

Lichen planus (choice C) is a dermatologic condition that may present with white oral plaques.
Microscopically,
the lesion is characterized by intense lymphocytic infiltration of the dermoepidermal (or mucosal-
submucosal)
junction with destruction of the basal layer of cells. This inflammatory condition has not been shown
to be
premalignant.

Oral thrush (choice D) is a superficial candidal infection, typically occurring in the


immunosuppressed, or very
young. Thrush is an infectious, non-neoplastic disease.

Squamous papilloma (choice E) is a benign human papillomavirus (HPV) infection of the oral
mucosa. Typically
associated with HPV genotypes 6 and 11, squamous papilloma only rarely progresses to squamous
carcinoma.

A 71-year-old white male has had polycythemia vera for 12 years. Throughout this period he was treated
by
phlebotomy alone, and has remained stable. Recently he has noted a dragging sensation in his
abdomen, and
physical examination reveals massive splenomegaly, palpable to 7 cm below the ribs. This finding
may represent
the conversion of polycythemia vera to which of the following hematological disorders?
52

A. Chronic myeloid leukemia

B. Hairy cell leukemia

C. Myelodysplastic syndrome

D. Myeloid metaplasia with myelofibrosis

E. Waldenström's macroglobulinemia

Explanation:

The correct answer is D. There are four types of myeloproliferative disorders: chronic myeloid
leukemia (CML),
polycythemia vera, myeloid metaplasia with myelofibrosis (MMM), and essential thrombocythemia
(which is very
rare). Both CML and polycythemia vera are characterized by hypercellular bone marrow producing
all three cell
lines. Over a period of years, the bone marrow may burn out, and the marrow space is replaced by
fibrosis. In
this circumstance, hematopoiesis (including production of neoplastic cells) moves to extramedullary
sites,
predominantly the spleen and liver. This process is called myeloid metaplasia with myelofibrosis.

Although both polycythemia vera and CML (choice A) can progress to MMM, polycythemia vera
does not
progress to CML.

Hairy cell leukemia (choice B), a leukemia often associated with massive splenomegaly, is derived
from a B
lymphocyte, not a myeloid stem cell. It would not progress to a myeloproliferative disorder.

Myelodysplastic syndrome (MDS; choice C) refers to myeloid disorders featuring maturation defects
and
ineffective hematopoiesis. MDS produces cytopenia and acute leukemias, whereas myeloproliferative
disorders
produce polycythemia and chronic leukemia.

Waldenström's macroglobulinemia (choice E) is a plasma cell dyscrasia that produces IgM in excess.
It derives
from plasma cells and, as such, is not a myeloproliferative disease.

A 30-year-old male presents to his physician for a pre-athletic physical exam. He has no complaints or
significant
past medical history. During auscultation of the heart, a faint murmur consistent with aortic
regurgitation is heard.
Examination of the musculoskeletal system shows decreased range of motion of the hips bilaterally
and of the
spine in rotation and forward bending. No scoliosis is seen. Examination of the eyes and mouth is
unremarkable.
His skin is smooth and dry. It is thought that he has a spondyloarthropathy. Which of the following
tests would
53

best differentiate this patient's disease from the other spondyloarthropathies?

A. HLA-B27

B. MRI of the femoral head

C. Rheumatoid factor

D. Spinal x-ray

E. Stool culture

Explanation:

The correct answer is D. This young man has an inflammatory arthropathy involving his hips and
spine. The
disease is ankylosing spondylitis, which typically occurs in young men. Ankylosing spondylitis
begins at the
sacroiliac crests, then moves upward in the spine, causing inflammation and destruction of the
posterior
elements of the vertebral bodies. The posterior aspects of the vertebrae fuse, reducing the range of
motion,
and partially taking the weight off the vertebrae, leading to an atrophy osteoporosis. The spinal x-ray
will show
fusion of the disks and possible intervertebral disc ossification. Patients may also have associated
inflammation
and fibrosis of the proximal aorta, leading to aortic regurgitation.

HLA-B27 (choice A) is a class I histocompatibility antigen that has a strong association with
ankylosing
spondylitis, Reiter's syndrome, psoriatic arthritis, and certain enteropathic arthritides. It is found in all
spondyloarthropathies and therefore can not help to distinguish between them.

MRI of the femoral head (choice B) would reveal inflammatory arthritis in that location, but that
would not be
enough to differentiate ankylosing spondylitis from the other entities. The spinal findings are more
characteristic
of this disease.

Rheumatoid factor (choice C) is typically negative for all of the spondyloarthropathies, as are other
serologic
markers of rheumatoid arthritis.

Stool culture (choice E) is not of diagnostic importance in distinguishing these entities. Ankylosing
spondylitis is
not associated with diarrhea or stool pathogens.

A patient consults a physician because of a small lesion on the lips that, on biopsy, proves to be a mucosal
neuroma. The patient's mother had medullary carcinoma of the thyroid. In addition to medullary
carcinoma of the
thyroid, to which of the following conditions would this patient be particularly vulnerable?
54

A. Gastrinoma

B. Insulinoma

C. Parathyroid adenoma

D. Pheochromocytoma

E. Pituitary adenoma

Explanation:

The correct answer is D. You should recognize this as a probable case of multiple endocrine
neoplasia,
specifically, MEN III (formerly MEN II b). Features of this autosomal dominant condition include
medullary
carcinoma of the thyroid, pheochromocytoma, and oral and intestinal ganglioneuromatosis (including
mucosal
neuromas).

Gastrinomas (choice A) and insulinomas (choice B) are found in MEN I.

Parathyroid adenomas (choice C) are found in MEN I and II.

Pituitary adenomas (choice E) are found in MEN I.

A 35-year-old woman complains of severe lower abdominal pain, which is worst during menstruation.
Laparoscopic examination of the pelvis demonstrates multiple small brown spots on the surface of
pelvic
structures. Most of these lesions are cauterized, but biopsy of one of the remaining lesions reveals
glandular
tissue resembling normal endometrium. No cytologic atypia or abnormally shaped glands are seen.
Which of the
following is the most likely diagnosis?

A. Acute endometritis

B. Adenomyosis

C. Chronic endometritis

D. Endometriosis

E. Metastatic endometrial cancer

Explanation:

The correct answer is D. This benign condition is endometriosis, rather than metastatic endometrial
cancer.
55

Endometriosis is defined as a benign growth of endometrium at sites at which it does not normally
occur
(excluding the myometrium, at which site it is called adenomyosis). Endometriosis is common and is
a significant
cause of both pain and pelvic scarring. Most of the problems occur because the abnormally located
endometrium responds to hormonal control and may menstruate, producing a very irritating fluid.
Endometriosis
may apparently be started either by seeding of menstrual cells in the pelvis (entering through the open
end of
the fallopian tubes) or by metaplasia of mesothelial or other cells (accounting for rare, well-
documented cases
of endometriosis of bizarre sites such as nasal mucosa or lungs).

Acute endometritis (choice A) is acute inflammation of the endometrium.

Adenomyosis (choice B) is characterized by foci of endometrium deep in the myometrium.

Chronic endometritis (choice C) is chronic inflammation of the endometrium.

Metastatic endometrial cancer (choice E) would be characterized by abnormal glands and cytologic
atypia.

A 32-year-old man presents to his physician for a routine physical examination. The man admits to recent
loss of
10 pounds and occasional fatigue, but ascribes these to increases in his workload. On physical
examination
there is a 2-3 cm firm, freely moveable, nontender mass in his neck on the right side. Biopsy of the
neck mass
reveals Reed-Sternberg cells in a mixed inflammatory infiltrate. CT scan reveals marked enlargement
of
mediastinal nodes and the presence of a single nodule in his liver. How should his disease be staged?

A. Stage IA

B. Stage IB

C. Stage IIA

D. Stage IIB

E. Stage IIIA

F. Stage IIIB

G. Stage IVA

H. Stage IVB

Explanation:

The correct answer is F. This is a classic presentation for Hodgkin's disease, a form of lymphoma
characterized
56

by neoplastic proliferation of Reed-Sternberg cells admixed with variable numbers of reactive


lymphocytes,
neutrophils, and eosinophils. About half the patients have usually painless adenopathy in the neck or
supraclavicular area at the time of diagnosis. A minority have constitutional symptoms such as
fatigue, weight
loss, and night sweats, but these can be important clues. Staging of Hodgkin's disease is based on the
extent
of spread and the presence or absence of constitutional symptoms. The man in question has
involvement of
cervical lymph nodes, mediastinal nodes, and the liver at the time of diagnosis, so his disease would
be stage III
(involvement of lymph nodes or extralymphatic organs on both sides of the diaphragm). The presence
of
constitutional symptoms makes this stage IIIB (if constitutional symptoms were absent, it would be
IIIA [choice
E]).

Stage I disease (choices A and B) is limited to a single lymph node region or a single extralymphatic
organ.

Stage II disease (choices C and D) can involve two or more lymph node regions on one side of the
diaphragm,
or can involve contiguous extralymphatic organs or tissues.

Stage IV disease (choices G and H) is defined by the presence of multiple or disseminated disease
foci in
extralymphatic organs or tissues. Lymphatic involvement may occur, but need not be present for the
diagnosis.

In which of the following sites is embryonal rhabdomyosarcoma most likely to occur?

A. Gastrointestinal tract

B. Head and neck

C. Kidneys

D. Liver

E. Lungs

Explanation:

The correct answer is B. Embryonal rhabdomyosarcoma is the most common form of


rhabdomyosarcoma and is
composed of cells resembling those found in developing muscle, predominately small, round cells. In
embryonal
rhabdomyosarcomas that protrude into an open space, the malignant cells immediately below the
surface may
be more densely packed, forming a "cambium" layer. The head and neck (particularly the nose,
nasopharynx,
57

and orbit) is the most frequent site for embryonal rhabdomyosarcoma; other favored sites are the
genitourinary
tract and the extremities.

In which of the following neurodegenerative conditions would you expect to observe the phenomenon
known as
anticipation?

A. Familial Alzheimer disease (FAD)

B. Familial amyotrophic lateral sclerosis (ALS)

C. Huntington disease

D. Pick disease

E. Progressive supranuclear palsy

Explanation:

The correct answer is C. Anticipation is a phenomenon in which the phenotype of a disease worsens
over
successive generations. This has been observed in families affected by a hereditary disorder because
of an
expansion of unstable sequences of nucleotide repeats (triplet repeat expansion). Clinical features
worsen with
each successive generation as the number of triplet repeats increases. Huntington disease is caused by
expansion of an unstable CAG repeat in a gene encoding a protein called huntingtin, of unknown
function. All
the unstable triplet-repeat disorders identified so far are associated with neurodegenerative conditions.
Other
examples are fragile X syndrome, myotonic dystrophy, and Friedreich ataxia.

FAD (choice A) comprises 5% to 10% of all cases of Alzheimer disease and is due to autosomal
dominant
mutations of three different genes: amyloid precursor protein (APP) gene, presenilin-1 gene, and
presenilin-2
gene. The e4 allele of the gene encoding apolipoprotein E increases the risk for FAD. Unstable repeat
expansion is not a cause of FAD.

The great majority of cases of ALS (choice B)are sporadic. A small subset of familial ALS is caused
by
mutations in the gene coding for superoxide dismutase on chromosome 21.

Pick disease (choice D) and progressive supranuclear palsy (choice E) are usually sporadic. The
genetic
alterations of the very few familial cases reported have not been elucidated.

A 37-year-old woman is in a serious automobile accident and sustains a closed head injury. She does not
58

immediately seek medical attention, but is brought to the emergency room two hours later by her
brother. On
physical examination, there is mydriasis and loss of the pupillary light reflex. Several hours later, she
is unable to
follow a flashlight with her eyes. Which of the following types of herniation is most likely occurring
in this patient?

A. Cerebellar tonsils into the foramen magnum

B. Cerebellum upward past the tentorium

C. Cingulate gyrus under the falx

D. Medulla into the foramen magnum

E. Temporal lobe under the tentorium

Explanation:

The correct answer is E. Head trauma can cause subdural or epidural hematomas that force the medial
aspect
of the temporal lobe (uncus) under the free edge of the tentorium cerebelli (an uncal herniation). The
resulting
compression of the oculomotor (III) nerve characteristically affects the peripheral areas of the nerve,
which
carry parasympathetic information, before affecting the central areas of the nerve, which carry
somatomotor
information. Recognizing the development of symptoms related to this damage may permit life-
saving medical
or surgical intervention before irreversible damage is done to the brainstem.

Tonsillar herniation into the foramen magnum (choice A) can compress the medulla, causing
respiratory failure
and death.

Upward herniation of the cerebellum (choice B) is occasionally seen in patients with posterior fossa
lesions.

Herniation of the medially located cingulate gyrus under the falx cerebri (choice C), also called
subfalcine or
cingulate herniation, can be seen with cerebral hemisphere expansion and can compress the anterior
cerebral
artery.

The medulla (choice D) is normally in the foramen magnum.

Following a fall on a basketball court, a 20-year-old man develops a swollen and painful upper arm. Over
the next
several weeks, the involved area becomes more circumscribed and firm, and then later evolves to a
painless,
hard, well-demarcated mass. X-ray of the arm at this point would most likely demonstrate which of
the following?
59

A. Benign-appearing bony outgrowths from the humerus

B. Dislocation of the shoulder

C. Flocculent radiodensities surrounding a radiolucent center

D. Healing fracture

E. Malignant-appearing bony outgrowths from the humerus

Explanation:

The correct answer is C. This is a typical presentation of myositis ossificans, which usually occur in
athletic
adolescents and young adults following trauma. Following muscle trauma, an area of damage heals
with a
fibroblastic proliferation that then ossifies, even though there is no connection to bone. Flocculent
radiodensities surrounding a radiolucent center are seen on x-ray. The lesion can microscopically
resemble
extraskeletal osteosarcoma, but the clinical history is usually quite different (osteosarcoma is a
disease of the
elderly). Simple excision usually cures myositis ossificans.

You might also like